18-preptest 18 explsdl.keywin.org/4/9/49d31fb5129d388a6dd62829ca482add.pdf · 2012. 4. 13. ·...

79
KAPLAN LSAT PREP LSAT RELEASED TEST XVIII EXPLAINED A Guide to the December, 1992 LSAT KAPLAN The answer to the test question.

Upload: others

Post on 11-Oct-2020

0 views

Category:

Documents


0 download

TRANSCRIPT

Page 1: 18-PrepTest 18 Explsdl.keywin.org/4/9/49d31fb5129d388a6dd62829ca482add.pdf · 2012. 4. 13. · wrong choices in a “could be false” question are statements that must be true.)

KAPLAN LSAT PREP

LSAT

RELEASED TEST XVIIIEXPLAINED

A Guide to the December, 1992 LSAT

KAPLANThe answer to the test question.

Page 2: 18-PrepTest 18 Explsdl.keywin.org/4/9/49d31fb5129d388a6dd62829ca482add.pdf · 2012. 4. 13. · wrong choices in a “could be false” question are statements that must be true.)

1995 Stanley H. Kaplan Educational Center Ltd

All rights reserved. No part of this book may be reproduced in any form, byphotostat, microfilm, xerography or any other means, or incorporated into anyinformation retrieval system, electronic or mechanical, without the written permissionof Stanley H. Kaplan Educational Center Ltd.

Page 3: 18-PrepTest 18 Explsdl.keywin.org/4/9/49d31fb5129d388a6dd62829ca482add.pdf · 2012. 4. 13. · wrong choices in a “could be false” question are statements that must be true.)

© K A P L A N 1

SECTION I:

LOGIC GAMES

Page 4: 18-PrepTest 18 Explsdl.keywin.org/4/9/49d31fb5129d388a6dd62829ca482add.pdf · 2012. 4. 13. · wrong choices in a “could be false” question are statements that must be true.)

LSAT PREP ______________________________________________________________ LSAT Test XVIII Explained: Section I

2 © K A P L A N

GAME 1 — Coeds in Canada(Q. 1-6)

The Action: In this game, we’re grouping five students—Hubert, Lori, Paul, Regina, andSharon—in the cities that each visits—Montreal, Toronto, or Vancouver. The Key Issueswill be:

1) What student visits what city?2) What student visits a city with what other student?3) How many cities can—or must—be visited?4) How many people can—or must—visit each city?

The Initial Setup: Let’s simply list the three cities on the page, so we can fill in the studentsthat go to each. (You’d probably do exactly that if, in real life, you were working out thestudents’ itineraries.) Remember to list the students off to the side:

H L P R S M T V

The Rules:

In the game intro we’re told that each student visits exactly one of the cities. Make a mentalnote to that effect or jot it down.

1) This rule becomes quite important when it comes time to work out the numbers aspectof this game. For now, just write “S ≠ P” or “NEVER SP” as a reminder of this rule.

2) Likewise, “H = R” will help us to remember this one.

3) Let’s build this right into our master sketch. Since Lori visits M or T, we can put an “L”above the sketch with arrows pointing down to the “M” and “T” columns.

4) “If P visits V, then H visits V” should do it for this rule. Note the contrapositive: “If Hdoesn’t visit V, then P doesn’t visit V.” And take it even further: “Doesn’t visit V” in thiscontext means “does visit Montreal or Toronto,” so the contrapositive really becomes“If Henry’s in M or T, then Paul is in M or T.” Just don’t get the impression that Paulwould necessarily have to travel with Henry—that’s only required if they go toVancouver.

5) None of the students visits a city alone. This is a very important rule. However youchoose to rephrase it, be sure to give it a prominent position.

Page 5: 18-PrepTest 18 Explsdl.keywin.org/4/9/49d31fb5129d388a6dd62829ca482add.pdf · 2012. 4. 13. · wrong choices in a “could be false” question are statements that must be true.)

LSAT PREP ______________________________________________________________ LSAT Test XVIII Explained: Section I

© K A P L A N 3

Key Deductions: Rule 5 needs some more thought. If all five of the students visit a citywith at least one other student, there’s only one possible breakdown for the fivestudents: 3 to one city and 2 to another. Rule 1 is so important because it prevents all fivestudents from visiting the same city.

We can go further with this thinking. H and R are always together, and S and P are alwaysapart. Therefore, the group of three will always consist of H, R, and either S or P. Byprocess of elimination then, the group of two will always consist of L and either S or P.No doubt, this game is doable without taking the information this far, but if you did workall this out in advance, it would greatly reduce the amount of time needed for thequestions.

The Final Visualization: And here is our neat, accessible master sketch:

H L P R S*No one alone*

3H R S/P

S ≠ PH = R

If P visits V, H visits VIf H visits M or T, P visits M or T

M T VL

2L S/P

The Big Picture:

• Avoid “caveman” talk. “If Paul visits Vancouver” means much more than “If P, V”or “P —> V.” And don’t oversimplify your scratchwork either. Writing down thatone word “visits,” if necessary, will help you remember and access the rule whenyou need it.

• Always attempt to work out the numbers in the game by asking yourself: “How can Ido this?”—which in this case means: “How can I get five people into three slotsgiven the constraints?” While you may be able to answer the questions without theextra numbers deduction, the game can be extremely time consuming withoutrealizing that you need to form one group of 2 and one group of 3.

• Time taken up front is very important. In general, you’ll do best if you take time todeduce as much as you can from the setup and rules before moving on to thequestions.

Page 6: 18-PrepTest 18 Explsdl.keywin.org/4/9/49d31fb5129d388a6dd62829ca482add.pdf · 2012. 4. 13. · wrong choices in a “could be false” question are statements that must be true.)

LSAT PREP ______________________________________________________________ LSAT Test XVIII Explained: Section I

4 © K A P L A N

The Questions:

1. (C)This doesn’t take the form of a normal “acceptability” question, but that’s really all it is.Use Kaplan’s technique for these questions, and compare each rule against each choice,eliminating those choices that violate a rule. Rule 1 is violated by (E). Rule 2 lets useliminate (A). Rules 3 and 4 are both violated by (D). Rule 5 takes care of (B) which leavesus with (C).

• This is an unusual way of presenting an acceptability question. But your experiencewith acceptability questions in many other games should have helped you recognizethis one as such as soon as you saw the nature of the answer choices.

2. (D)Rule 2 dictates that our group of three is H, R, and S, which makes our group of two L andP. And that’s really all we need to check the choices. P and L can visit Toronto while H, R,and S go to Montreal or Vancouver, so (D) can be true.

(A), (B), and (C) are all eliminated by the simple fact that the big group is H, R, S, and thelittle group is P, L.

(E) No, Paul must be with Lori, which means he’s in Montreal or Toronto. If he visitsVancouver, Rule 4 forces Hubert, not Lori, to accompany him.

• In some grouping questions, the main issue is who’s with whom, and we’re not asconcerned with where they actually go. Here, there is certainly flexibility as to whichgroups are in which cities, but the groups themselves are absolutely determined,which lets us eliminate most of the choices.

Page 7: 18-PrepTest 18 Explsdl.keywin.org/4/9/49d31fb5129d388a6dd62829ca482add.pdf · 2012. 4. 13. · wrong choices in a “could be false” question are statements that must be true.)

LSAT PREP ______________________________________________________________ LSAT Test XVIII Explained: Section I

© K A P L A N 5

3. (D)If Sharon goes to V, then Paul can’t visit V (Rule 1), and neither can Lori (Rule 3), so Hubertand Regina must join Sharon in Vancouver. Even if you hadn’t made the extra numbersdeduction, Rule 5 would still tell you that Paul and Lori must team up to go to one of theother cities, which is choice (D).

(A) Hubert visits Vancouver, not Montreal.

(B) Lori could visit Montreal, but she could just as easily go to Toronto.

(C) Paul could visit Toronto, but he could just as easily go to Montreal.

(E) No, Lori is teamed up only with Paul, while Regina is with Hubert and Sharon.

• Work with the new information to glean as much as possible, but when you hit alull, go to the answer choices. Knowing when it’s best to stop looking for newdeductions and to move on to the choices is a skill that will develop with practice.

4. (A)Not much to do here except check the choices. Let’s start with (A). The operative questionis: Could Sharon go somewhere besides M if Paul visits V? Rules 4 and 2 force the group ofthree to be Paul, Hubert and Regina visiting Vancouver. Can Sharon go somewhere otherthan M? Yes, she and Lori can go to Toronto. (A), therefore, could be false and is theanswer.

(B) If Paul visits V, Rule 4 forces H to join him, and Rule 2 forces Regina to also visitVancouver.

(C), (D), and (E) These choices all reflect the extra deductions we made earlier. Reginavisits a city with Hubert (Rule 2) and either Sharon or Paul (Rule 1). This leaves Lori pairedwith exactly one other student, either Sharon or Paul, whichever doesn’t join the groupwith Regina and Hubert.

• If you hadn’t made the extra deduction about the groups of students, here’s whereyou could have picked up on it. Three of the four wrong answer choices hint at it.

• In testing to see whether a choice could be false, look for an exception to the choice.If you do find an exception to what the choice asserts (Sharon MUST visit Montreal),then that choice could be false, and is the answer you seek. (And remember—all thewrong choices in a “could be false” question are statements that must be true.)

• Have confidence in your work! If you tried out choice (A) and came to the logicalconclusion that Sharon need not go to Montreal under these circumstances, andtherefore that the statement could in fact be false, there’s no reason to waste valuabletime on the remaining choices.

Page 8: 18-PrepTest 18 Explsdl.keywin.org/4/9/49d31fb5129d388a6dd62829ca482add.pdf · 2012. 4. 13. · wrong choices in a “could be false” question are statements that must be true.)

LSAT PREP ______________________________________________________________ LSAT Test XVIII Explained: Section I

6 © K A P L A N

5. (C)Hubert and either S or P will join Regina in Toronto (Rules 2 and 1), so that leaves Lori andeither S or P to visit Montreal. Paul could be in the group of three in Toronto, if Sharon iswith Lori in Montreal—(C) could be true.

(A) All five students can’t visit the same city (Rule 1). The group of three is in Toronto, soLori (who’s in the group of two) must go to Montreal.

(B), (D), and (E) The only two cities visited in this scenario are Toronto and Montreal.Cross off any choices that include a student visiting Vancouver.

• Work with the new information, and see how it interrelates with what you alreadyknow.

6. (E)Not much to do except check the choices, crossing off anything that need not be true. In theend, we’re left with (E). And (E) represents an idea we’re very familiar with by now—thegroup of two, including L and either S or P, must visit Montreal or Toronto, thanks to Rule3, which means that if anyone goes to Vancouver, it must be in the context of that group ofthree (H, R and S or P).

(A), (B), and (C) H, R, and [S or P] could visit Montreal while L and [S or P] visit Toronto,which kills the first three choices.

(D) The stem of Q. 3 had Sharon visiting Vancouver, and Rule 1 prohibits Sharon and Paulfrom visiting the same city.

• Usually a “must be true” with no new information is a signal that a Big Deduction isbeing tested. However, since all five of the choices are in “if/then” form, you shouldhave realized that this question would involve testing out choices, which couldbecome a time-consuming affair.

• Use your previous work whenever possible, and pay special attention to previousquestion stems when looking for things that can or cannot be true. This is especiallyhelpful when eliminating choices in “must be true” or “cannot be true” questions.

Page 9: 18-PrepTest 18 Explsdl.keywin.org/4/9/49d31fb5129d388a6dd62829ca482add.pdf · 2012. 4. 13. · wrong choices in a “could be false” question are statements that must be true.)

LSAT PREP ______________________________________________________________ LSAT Test XVIII Explained: Section I

© K A P L A N 7

GAME 2 — Course Folders(Q. 7-13)

The Action: This game may look complex, but the confusion stems mainly from the natureof the entities themselves. Once we understand that we’re dealing with six entities, eachcharacterized by two elements (season and subject), it becomes clear that this is really justanother sequencing game. Three courses are each offered twice—once in the spring, andonce in the fall—so we have six entities total: fall mathematics, spring mathematics; fallnutrition, spring nutrition; fall oceanography, spring oceanography. For our purposes,let’s abbreviate them as fM, sM, fN, sN, fO, and sO. Each of the six numbered folderscontains an order form from one of the six entities. The Key Issues are similar to thosefound in any standard sequence game:

1) What course’s order form is in what folder?2) What course’s order form can, must, or cannot be in a consecutive folder with whatother course’s order form?

The Initial Setup: It’s a good idea to keep it simple: Put the numbers 1 through 6 acrossthe top to represent the folders, and the six entities off to the side:

fM fN fOsM sN sO

1 2 3 4 5 6

The Rules:

1) A basic loophole-closer; no folder contains more than one course’s order forms.

2) It’s important to distinguish between subject and season. This rule states that the subjectof folder 1 is the same as that in folder 2. In other words, if fall mathematics is in folder 1,then spring mathematics must be in folder 2. One way to indicate this is by writing “samesubject” above the 1 and 2 in our master sketch.

3) Here we’re told that the subject in folder 3 is different from the subject in folder 4— “diffsubject” above 3 and 4 will take care of this.

4) and 5) both concern folders 1 and 4: Fall mathematics is in 1 or 4, and springoceanography is in 1 or 4. Obviously then, whichever orders are in 1, the other orders willbe in 4. We can build this right into our sketch by writing “fM or sO” over the 1 and 4.

6) “No sN” over the 5 should remind us of this one.

Page 10: 18-PrepTest 18 Explsdl.keywin.org/4/9/49d31fb5129d388a6dd62829ca482add.pdf · 2012. 4. 13. · wrong choices in a “could be false” question are statements that must be true.)

LSAT PREP ______________________________________________________________ LSAT Test XVIII Explained: Section I

8 © K A P L A N

Key Deductions: Let’s take a minute to play out the options related to the fM/sO ordersin folders 1 and 4. If fM is in 1, then sO goes in 4. Rule 2 then forces sM into 2. Both mathfolders are taken care of, and with sO in 4, the other oceanography course can’t be in 3(Rule 3), so folder 3 must contain nutrition, although we don’t know which one. This leavesthe other nutrition and fall oceanography to float between 5 and 6 (as long as sN isn’t infolder 5 ). We’ll call this option 1.

Now let’s try sO in 1 and fM in 4. Now Rule 2 forces fO into 2, and we get the samesituation: Oceanography is finished, Math can’t go in folder 3 (Rule 3), so nutrition must bein folder 3. This time, the other nutrition and sM will float in the 5 and 6 spots (again, aslong as sN isn’t in folder 5). This scenario will be option 2.

The upshot of this analysis is that a nutrition order form must be in folder 3, and thatevery situation in every question can be plugged into one of these two options, whichshould make our work much easier.

The Final Visualization: When we’re able to isolate such specific options, they’re all weneed to answer the questions. The options described above look like this:

Option #1: fM sM N sO fO/N fO/NOption #2: sO fO N fM sM/N sM/N

1 2 3 4 5 6

The Big Picture:

• You have to be able to think on your feet on test day if and when slightly unusualfeatures pop up in a game. Here, instead of the usual convention for the entities (Athrough F, L through Q , for example), each of the six entities is defined by twothings, instead of one. As long as you don’t freak out about a subtle change like this,you should be able to successfully adapt and incorporate the slightly “new” featureinto your work. But even this feature isn’t all that new—this entire game is verysimilar to the “Hats on Hooks” game in the Kaplan Lesson book, right down to thedoubly-named entities. Both games even have two possibilities that you can workout ahead of time.

• When an entity is restricted to only two possible options, it’s worth testing out whathappens in each case. When two entities in the game are so restricted, it’s a good betyou’ll be able to significantly determine the major possible scenarios in advance.And when you can do this, the original master sketch becomes superfluous, aseverything you need to know about each entity is contained within the pre-configured options.

• If you’re having trouble understanding a rule, formulate an example that will help toclarify the rule and to engrave it in your mind.

• The nature of the questions often give away the level of up front deduction that’spossible in a game. It’s no coincidence that we could deduce so much and that fiveout of the seven questions are non-if questions.

Page 11: 18-PrepTest 18 Explsdl.keywin.org/4/9/49d31fb5129d388a6dd62829ca482add.pdf · 2012. 4. 13. · wrong choices in a “could be false” question are statements that must be true.)

LSAT PREP ______________________________________________________________ LSAT Test XVIII Explained: Section I

© K A P L A N 9

The Questions:

7. (D)No different from any acceptability question—just check the rules against the choices. Rule2 kills (B). Rule 3 takes care of (E). Rule 4 knocks off (C). Rule 5 axes (A), leaving the answeras (D).

• “Acceptability” questions are our friends; they help to cement the concept of a game.And if you had trouble understanding the basic action of the game, the very form ofthis question and its choices should help to clear things up.

8. (A)This comes straight off of our deduction, made clear by the two options above, that folder3 has to contain nutrition orders.

(B) Indeed, we deduced that either the fall or spring nutrition orders are in folder 3.

(C) Our second option (and Rule 5, for that matter), shows this to be a definite possibility.

(D) and (E) Option 1, fleshed out like so, takes care of these choices: 1—fM, 2—sM, 3—fN,4—sO, 5—fO, 6—sN.

• When you’ve made deductions up front, actively pursue answers to non-ifquestions.

9. (B)Fall oceanography in 2 tells us that we’re dealing with our second option, so we know thatsO is in 1 and fM is in 4. We move on to the choices, looking for one that works (could betrue). The spring mathematics orders could be in 6 as long as the spring nutrition ordersare in 3 and the fall nutrition orders are in 5. (B) is possible, and is our answer.

(A) No, the fall mathematics orders are in 4.

(C) No, the spring oceanography orders are in 1.

(D) Rule 6 forbids the spring nutrition orders from being in 5, so they must be in either 3or 6.

(E) We deduced early on that either the spring or fall nutrition orders must be in 3.

• Use the questions to increase your mastery of the game—if you hadn’t picked up onthe “sN or fN in 3” deduction, you might have seen it here.

Page 12: 18-PrepTest 18 Explsdl.keywin.org/4/9/49d31fb5129d388a6dd62829ca482add.pdf · 2012. 4. 13. · wrong choices in a “could be false” question are statements that must be true.)

LSAT PREP ______________________________________________________________ LSAT Test XVIII Explained: Section I

10 © K A P L A N

10. (D)Another non-if could be true question, so we have no choice but to look through thechoices. The previous question (as well as option 2) conforms perfectly to (D). Thisscenario was possible then, and it’s certainly possible now.

(A), (B), (C) Just the basics here—we know from our two options (or from Rules 4 and 5)that either fall mathematics or spring oceanography is in folder 1, and that one of those is infolder 4 as well.

(E) The same old deduction kills this one—one of the nutrition order forms must be in 3.

• Use your previous work whenever possible. By the fourth question of a game, youshould be familiar with where the entities are likely to be placed.

• Don’t be surprised if some questions fall fairly quickly, especially when you’veworked out a lot of things in advance. The whole point of doing such work up frontis to greatly reduce the time required to answer the questions.

11. (B)We did this work in figuring out the second option. We’re given fO in 2, so sO must be in 1and fM must be in 4. That’s all we know for sure (the rest could float between folders 3, 5,and 6 so long as there’s an N in 3 and sN isn’t in 5), so that makes two of the remainingfolders that can be deduced, choice (B).

• Read carefully throughout the entire Logic Games section. If you missed the phrase“of the remaining folders” and included fall oceanography in your count, you wouldhave counted one folder too many.

12. (E)Folders 5 and 6 contain nutrition and oceanography in option 1 and nutrition and math inoption 2. No matter how we slice it, 5 and 6 must contain different subjects. You may haveseen this simply from consulting the options, or you could have eliminated the wronganswer choices, like so:

(A), (D) Option 2: Spring nutrition orders could be in 3, and fall nutrition orders in 5,leaving the math orders for 4 and 6.

(B), (C) Also in option 2, fall mathematics is in 4, and spring mathematics could be in 5,leaving the two nutrition orders for 3 and 6.

• Sometimes you have to rephrase the question stem in order to make it moremanageable. This one may be easier if translated into “which folders can’t have thesame subjects?”

Page 13: 18-PrepTest 18 Explsdl.keywin.org/4/9/49d31fb5129d388a6dd62829ca482add.pdf · 2012. 4. 13. · wrong choices in a “could be false” question are statements that must be true.)

LSAT PREP ______________________________________________________________ LSAT Test XVIII Explained: Section I

© K A P L A N 11

13. (C)In eliminating the wrong choices in Q. 12, we just saw how spring mathematics could be infolder 6, so choice (C) could indeed be true. The entire order was 1—sO, 2—fO, 3—sN, 4—fM, 5—fN, 6—sM. Once again, you probably had to tackle this by quickly checking eachchoice against the two options.

(A) If there’s one thing we know by now, it’s that these two courses can’t be consecutive—one of them is in 1 and the other is in 4.

(B) The only other spring courses, of course, are oceanography and nutrition. Both optionsassign spring oceanography to a folder other than 5, and Rule 6 explicitly states that springnutrition can’t go there, so this can’t be true.

(D) Impossible—fall mathematics orders are in either 1 or 4 with spring oceanography inthe other.

(E) This would force us into option 1, with fall mathematics in 1, fall nutrition in 3, and falloceanography in 6. That option has spring mathematics in 2, and spring oceanography in 4.The problem is that this leaves spring nutrition for folder 5, which is a no-no thanks to Rule6.

• Trying out choices could be time consuming, but by the final question of a game,you should be very used to where the entities “like” to go. Most of these choices areexpendable for fairly obvious reasons; they either directly violate rules or blatantlycreate situations we know can’t exist.

• This game has seven questions, five of which are non-ifs. This is the type of gamethat you should seek out and tackle early on, no matter where it appears in thesection.

Page 14: 18-PrepTest 18 Explsdl.keywin.org/4/9/49d31fb5129d388a6dd62829ca482add.pdf · 2012. 4. 13. · wrong choices in a “could be false” question are statements that must be true.)

LSAT PREP ______________________________________________________________ LSAT Test XVIII Explained: Section I

12 © K A P L A N

GAME 3 — Subway Lines(Q. 14-19)

The Action: This type of logic game—a mapping game—has appeared on the testoccasionally throughout the years. Although you may not see one of this type on your test,you just might. They’re certainly worth practicing, if for no other reason than they respondto the same methods needed to solve other more “regular” types. In this one, five subwaylines—L1, L2, L3, L4, and L5—connect a total of eight stations. The Key Issues will be:

1) What stations are accessible by what lines?2) What stations are accessible by more than one line, making transfers possible?3) What stations and lines can, must, or cannot be used when traveling to the variousstations?

The Initial Setup: In a mapping game, you should try to build all of the rules into asingle, precise, accessible sketch. You’ll want to do this by combining the various routes, inthis case subway lines, into the same sketch. It’s best to settle on a convention that will helpyou to distinguish one line from the other, like so:

L1 = ——L2 = ~~~~L3 = L4 = x x x xL5 = º º º º

The Rules:

An important rule is buried in the setup paragraph: We don’t have to worry about whichdirection the lines run, because the trains run in both directions, stopping at every station.Did you miss that in your first attack on the game?

1) L1 runs in a loop, and connects the most stations, so it makes the most sense to draw thisline as the outside of the diagram. Be sure to make it big enough to accommodate all of theother lines. For our purposes we’re using a solid line to represent L1. Check to make sureyou got the order of the stations right; it’s a loop that begins and ends with Rincon.

2) We chose a squiggly line for L2, so connect the T and S, already included in L1, with asquiggly line.

3) We’re using a dashed line for L3, so connect R and U with a dashed line.

Page 15: 18-PrepTest 18 Explsdl.keywin.org/4/9/49d31fb5129d388a6dd62829ca482add.pdf · 2012. 4. 13. · wrong choices in a “could be false” question are statements that must be true.)

LSAT PREP ______________________________________________________________ LSAT Test XVIII Explained: Section I

© K A P L A N 13

4) This rule introduces a new station, Greene. We’re using a line of Xs for L4, so we need todraw a line of Xs from Q to our new G and on to R.

5) A line of circles represents L5, so draw such a line from Q to T. And that completes ourimpromptu subway map.

Key Deductions: While there may not be any earth-shattering deductions here, it isworthwhile to take a few seconds to notice certain things highlighted by the master sketch.For instance, F, G, and P are the only stations connected to only one train line, which limitsthe options for a train heading to or through any of these stations.

The Final Visualization: Here’s our creation, in all its glory:

L1 = ——L2 = ~~~~L3 = L4 = x x x xL5 = º º º º

R

P S

FTo

oo

oo

oo

oo

oo

oo

oo

Q U

x x x x x x x x x x x x x x x

~~~~~~~~~~~G

The Big Picture:

• Don’t worry if it takes you a little extra time in a mapping game to get to thequestions. In a mapping game, the map itself is the source for the answer to everyquestion, which is why it is so vital that the map is precise, accurate, and neat. Takethe time you need to make all the right connections, and you’re sure to be rewardedwith at least a few quick and easy points.

• Not all rules are in indented form. Some very important rules (like “trains run inboth directions, stopping at every station” ) are hidden in the opening paragraph.Make sure you don’t miss them.

Page 16: 18-PrepTest 18 Explsdl.keywin.org/4/9/49d31fb5129d388a6dd62829ca482add.pdf · 2012. 4. 13. · wrong choices in a “could be false” question are statements that must be true.)

LSAT PREP ______________________________________________________________ LSAT Test XVIII Explained: Section I

14 © K A P L A N

The Questions:

14. (C)Just look for the stations that are directly connected to R. L1 connects R with T and R withP; L3 connects R with U, and L4 connects R with G. That’s a total of four stations, choice(C).

• Don’t be surprised if after a lengthy setup, the first question is relativelystraightforward. Much like with an acceptability question, the testmakers just wantto make sure that you got through the setup unscathed.

15. (C)We have to get our weary traveler from G to S, making the fewest number of stops andusing the fewest number of lines. One line won’t get us there (L4 stops at R or Q), but howabout two lines? Yes; G to Q on L4, transfer to L1 to U and on to S. That’s two lines and twostops in between G and S. Can we do better? No, if we went G to R to T to S, that wouldalso be two stops in between, but it would require three different lines. So we know thetraveler must stop at Q and U in order to satisfy the conditions. Either could be correct; thetestmakers chose Q, choice (C).

(A), (B), (D), (E) We’ve seen the shortest way, with the fewest number of lines, and itdoesn’t include any of these stops.

• When asked for the “fewest” possible, start with the smallest number and work yourway up. Knowing that it’s possible to go from G to S with four stations in betweendoesn’t help you when it’s also possible with fewer stops.

16. (A)Our dotted line (L3) is out of commission, and we want the most direct route (i.e. feweststops) from U to R. Let’s look at the diagram. We could go U—Q—P—R, U—S—T—R, U—Q—G—R, or U—Q—T—R. That’s it. Every other way would include more than twointermediate stops. So the possible intermediate stops, in order, for this journey, are Q—P,S—T, Q—G, and Q—T, the latter of which is choice (A).

(B) We’d have to add T to this list to get to R.

(C) U—S—P—R is impossible (remember, in sequence).

(D) U—Q—P—G—R is impossible.

(E) This route would get us there, but not with the fewest number of intermediate stops.

• Always work out as much as you can before moving on to the choices. Here, you’rebetter off figuring out all the possible routes first, noticing the intermediate stops,and then simply looking for them among the choices.

Page 17: 18-PrepTest 18 Explsdl.keywin.org/4/9/49d31fb5129d388a6dd62829ca482add.pdf · 2012. 4. 13. · wrong choices in a “could be false” question are statements that must be true.)

LSAT PREP ______________________________________________________________ LSAT Test XVIII Explained: Section I

© K A P L A N 15

17. (C)To get from F to G, the train has to first stop at either S or T. There are four ways to gothrough S first: F S U Q G; F S U R G; F S T Q G; and F S T R G. Each of these routesrequires three intermediate stops. However, if the train leaves F and hits T first, it can go FT R G or F T Q G. Here, the number of intermediate stops is two, which proves to be theminimum.

• Read the question stem carefully. If you had done the work required but thencounted the total number of stops instead of the intermediate stops, your workwould’ve been wasted.

18. (E)With the U to Q segment down, check for the shortest way from S to P. We’ve got S—T—R—P, S—T—Q—P, and S—U—R—P. Simply checking these against the choices shows that(E) must be true.

(A) and (C) The S—U—R—P route shows that both F and T can be omitted, and that bothQ and T can be left off as well.

(B) and (D) The S—T—R—P possibility shows that we can get by just fine without G or U,and without Q or U.

• Ask yourself the relevant questions. When a question asks if it a train must passthrough F or T, for instance, the relevant question is: Can I leave both of those out? Ifso, then you’ve proven that the train need not go through F OR T; that is, it canbypass both, and you can cross that choice off.

19. (A)We thought through this point up front, while briefly studying our newly-minted map. Weidentified F, G, and P as the only stations connected to only one line. Therefore, the newline must include F, G, and P, choice (A).

All of the wrong choices include at least one station that’s already connected to more thanone line, and therefore need not be included in the new line.

• Be sure you understand exactly what a question is asking for. Only then can youeliminate those entities that already fulfill the new requirement. One way to do thisis to “unpack” the stem; that is, translate it into something that’s easier to work with.In this case, the question is simply looking for a choice that contains a complete andaccurate list of the stations connected to only one other station.

Page 18: 18-PrepTest 18 Explsdl.keywin.org/4/9/49d31fb5129d388a6dd62829ca482add.pdf · 2012. 4. 13. · wrong choices in a “could be false” question are statements that must be true.)

LSAT PREP ______________________________________________________________ LSAT Test XVIII Explained: Section I

16 © K A P L A N

GAME 4 — Law Firm Promotions(Q. 20-24)

The Action: This is one of the earliest examples of the game type that would evolve intowhat we now know as a “process” game: a game in which what happens in one stepdirectly affects what can happen in the next. At Kaplan, we anticipated this type of game,and even have a game in the Preview/Classroom/Review book (“University Promotions”)that’s similar. Here, a law firm initially consists of partners (Harrison and Rafael), anassociate (Olivos), and assistants (Ganz, Johnson, Lowry, Stefano, Turner, and Wilford). Allnon-partners are reviewed for promotion once a year. The Key Issues are:

1) What associates and assistants can, must, or cannot be promoted following the firstannual review?2) What associates and assistants can, must, or cannot be promoted following eachsubsequent annual review?

The Initial Setup: As with most process games, you shouldn’t overdepend on yourpencil, especially in the step-by-step question work. What you can do is get down theinitial placement of all the members of the law firm. It may be helpful to put the partnerson top, followed by the associate(s), with the assistants under them both. This gives you aphysical reminder that everyone is striving to be promoted up the ladder:

Part: HRAssoc.: OAssist.: G J L S T W

The Rules:

As well as giving the initial placements for all of our entities, the first paragraph givesother vital rules: At least one associate and one assistant are promoted following eachreview. A promotion is to the next highest rank, which means that an assistant can’t bepromoted directly to partner. An assistant is promoted when a majority of associates andpartners vote for his promotion, and an associate is promoted when a majority of partnersvote for her promotion. Everyone eligible to vote, votes. No one new joins the firm, and noone leaves the firm. These rules form the basis of understanding the entire game. If youglossed over the opening paragraph, this game was most likely a complete mystery.

As for the indented rules:

1) The only initial associate (Olivos) always votes thumbs down to Ganz, Johnson, andTurner.

2) Rafael (a partner) always votes nay to Lowry and Stefano.

Page 19: 18-PrepTest 18 Explsdl.keywin.org/4/9/49d31fb5129d388a6dd62829ca482add.pdf · 2012. 4. 13. · wrong choices in a “could be false” question are statements that must be true.)

LSAT PREP ______________________________________________________________ LSAT Test XVIII Explained: Section I

© K A P L A N 17

3) Harrison, the other partner, always votes no to Johnson and Wilford.

You can either rewrite each of these rules in shorthand, or simply circle them on the page.You’ll be referring back to them throughout the game, so do whichever you’re morecomfortable with.

Key Deductions: There’s something to notice right away just by looking at the initialnumber of lawyers in each level. The opening paragraph tells us that at least one associateand one assistant are promoted following each review. Since Olivos is initially the onlyassociate, then Olivos must be promoted to partner following the first review. Since noone can jump from assistant to partner in one review, this means that following the firstreview, the partners must be Harrison, Olivos, and Rafael, which helps considerably withacceptability Q. 20.

What else should you have noticed from the setup? The three rules deal with the only threepeople in the firm who initially have the right to vote. A majority of them must vote for anassistant in order for him or her to be promoted. Therefore, if any two or more of the three(Olivos, Rafael, and Harrison) vote against an assistant, then that assistant cannot bepromoted following the first review. Scanning the rules, we see that Olivos and Harrisonalways vote against Johnson. This tells us that Johnson cannot be promoted following thefirst review (which also helps with Q. 20).

The Final Visualization: Again, there’s not much to draw, but here it is, including thedeductions above:

Part: HRAssoc.: O O Promoted 1st ReviewAssist.: G J L S T W J not Promoted 1st Review

The Big Picture:

• If you depend solely on drawing for your Logic Games success, you are bound to bedisappointed. “Process” games are just one type that doesn’t lend itself todiagramming. Depend on sound thinking skills instead; those will never let youdown.

• The way the testmakers lay a game out on the page hints at the way they feel the gamecan and should be handled. If this game required lots of drawing and scratchwork,they probably would have had to provide more free space on the page. The fact thatthey didn’t is a clue that thinking, not scribbling, is what’s in order here. (Conversely,notice how much free space is on the page of the preceding game—here, it’s a goodbet that the testmakers are encouraging you to condense the five subway lines intoone manageable visual sketch.)

Page 20: 18-PrepTest 18 Explsdl.keywin.org/4/9/49d31fb5129d388a6dd62829ca482add.pdf · 2012. 4. 13. · wrong choices in a “could be false” question are statements that must be true.)

LSAT PREP ______________________________________________________________ LSAT Test XVIII Explained: Section I

18 © K A P L A N

• Not all rules are in indented form. Some extremely important rules are hidden in theopening paragraph.

• Previewing a section is extremely important. Even if this game was first instead offourth, you would have been well advised to leave it for third or fourth, after youhad accumulated points and confidence with the other games.

The Questions:

20. (E)Don’t be fooled by the physical layout of this one—it’s nothing more than an acceptabilityquestion. Check the rules (don’t forget the ones in the opening paragraph) and deductionsabout Olivos and Johnson against each choice. Olivos, and only Olivos is promoted topartner, which narrows it down to (A), (D), and (E). Johnson can’t be promoted in thisyear’s review; as we saw above, Johnson just doesn’t have the numbers the first timearound, so we can cross off (A). But someone has to go from assistant to associate, so we canalso kill (D), which leaves us with correct choice (E).

• “Acceptability” questions are excellent tools for getting a good grasp of what’s goingon in a game. Look to them, as well as to non-if “must be true” questions, to solidifyyour game conception and pick up what should be quick and easy points. Andspeaking of non-ifs . . .

Note: “Section Management” involves the art of searching out and doing the easiergames first, while leaving the toughest game (according to you, anyway) for last. “GameManagement” involves doing a similar thing within each game—skipping the toughquestions and racking up points wherever you can. Granted, in most games, you won’tfind yourself skipping randomly around when tackling the questions. But in a gamelike this, after finishing the acceptability question, you should notice something aboutthe remaining four questions on the page: The first two are rather long (especially Q. 21)and include hypotheticals, whereas the last two are fairly short non-if questions. Youmay have benefited from tackling Qs. 23 and 24 first, which, like the acceptabilityquestion, really only test a basic understanding of the game, as opposed to Qs. 21 and22, which require you to not only understand the setup but to apply new situations to it.If you worked through the last two questions first, you may have found that you pickedup three of the five questions on a tough game simply from an understanding of thesetup, which is pretty good, leaving you confident to try Qs. 21 and 22. Of course, thebottom line is the same doctrine that underlies the entire Kaplan approach: you have thepower to take control of the test; if you don’t do so, the test will take control of you.

Now back to our regularly scheduled program . . .

(For the sake of clarity in this document, we’ll continue with the questions in the order theywere presented.)

Page 21: 18-PrepTest 18 Explsdl.keywin.org/4/9/49d31fb5129d388a6dd62829ca482add.pdf · 2012. 4. 13. · wrong choices in a “could be false” question are statements that must be true.)

LSAT PREP ______________________________________________________________ LSAT Test XVIII Explained: Section I

© K A P L A N 19

21. (D)We’re looking for a list of associates resulting from this year’s review. Knowing what weknow, we can already eliminate (B) and (E): Johnson’s going nowhere this year, and Olivosis destined for partner. We’re given the only people that Rafael and Harrison vote for, sousing Rule 1 (which tells us who Olivos votes against), we’ll need to see which assistants inthe remaining choices can possibly garner two yes votes from this group of three initialvoters. Only Rafael votes for Ganz, so cross off (A). No one votes for poor Turner, so axe(C). We’re left with (D), which, by process of elimination, must be correct.

• When you’re eliminating choices, don’t bother to check the last one. Trust your workand move on. Having confidence in your abilities is key, and is the surest way toincrease your speed on the Logic Games section.

22. (B)At best, Johnson has two no’s and one yes from the original three voters. To get a majority ofyesses by the next vote, Johnson would need a total of three yesses. Therefore, at least twoassistants must be promoted in this review in order for Johnson to be promoted during thenext review, choice (B).

• This question illustrates the key skill required in process games—the ability toanticipate the factors that need to conspire in one phase of the process in order for aspecific result to occur in another.

23. (E)After two reviews what must be true? Well, at least two people must have joined theoriginal two partners, resulting in at least four partners. That’s not a choice. What else?Well, at least two assistants must have been promoted (at least one this year and one nextyear), leaving a maximum of four assistants left—and that is a choice, choice (E).

(A) Lowry could be an associate or even a partner by then.

(B) Actually, Wilford could still be languishing as an assistant.

(C) This only could be true. (Actually, since no one new joins the firm this could only betrue if the firm decided to discontinue the annual reviews, or go out and hire some newassistants. Remember, at least one assistant must be promoted after each review.)

(D) No, five of the assistants could have moved on by then.

• Actively pursue the answer in such non-if questions, but if nothing comes to you,there’s nothing wrong with trying out the choices.

• In process games, always pay attention to the distinction between the various stages.Keep the time frames clear in your mind—are they talking about this year, last year,next year, two years from now? and so on.

Page 22: 18-PrepTest 18 Explsdl.keywin.org/4/9/49d31fb5129d388a6dd62829ca482add.pdf · 2012. 4. 13. · wrong choices in a “could be false” question are statements that must be true.)

LSAT PREP ______________________________________________________________ LSAT Test XVIII Explained: Section I

20 © K A P L A N

24. (B)Here’s a nice gimme. Think in terms of a minimum: If this year’s review results in exactlyone associate, then in next year’s review, there’s no problem with having only one assistantpromoted to associate to fill the space of that one associate promoted to partner. If that’stoo abstract, try an example: This year, Olivos moves on to partner, so let’s say Ganz, andonly Ganz, takes his place as associate. Then, in next year’s review, as Ganz moves topartner, it’s fine for one associate, let’s say Lowry, to fill Ganz’s spot as associate. Whetheryou worked it out abstractly, or picked an example to show you the way, the smallestnumber of associates necessary after next year’s review is one, choice (B).

• Don’t make your work any harder than necessary. Sometimes, even the last questionof a game simply tests whether you understand the action and have a basic grasp ofthe rules.

• This question highlights what we were talking about above regarding timemanagement. This was the last question in the entire section, but it was one of thesimplest.

Page 23: 18-PrepTest 18 Explsdl.keywin.org/4/9/49d31fb5129d388a6dd62829ca482add.pdf · 2012. 4. 13. · wrong choices in a “could be false” question are statements that must be true.)

© K A P L A N 21

SECTION II:

LOGICAL REASONING

Page 24: 18-PrepTest 18 Explsdl.keywin.org/4/9/49d31fb5129d388a6dd62829ca482add.pdf · 2012. 4. 13. · wrong choices in a “could be false” question are statements that must be true.)

LSAT PREP _____________________________________________________________ LSAT Test XVIII Explained: Section II

22 © K A P L A N

1. (D)Parent 2 says “I disagree,” but with what? Not with the phenomenon of kids 9-11 dating—both parents agree that it’s happening—but with #1’s explanation of same. #1 blamesearly-childhood libido, while #2 blames peer pressure. In other words, #2 provides analternative explanation.

(A) The only comparison made here is not between “phenomena,” but between kids tenyears ago and kids today. And anyway, that comparison is made by Parent 1, not Parent 2.

(B) “An exceptional case” means just that: a single incident or individual that’s cited as anexception. Parent 2 makes no such reference.

(C) Since Parent 2 only discusses the 9-11 age group, we don’t know what he believes orassumes about the 13-15 age group.

(E) essentially accuses Parent 2 of an ad hominem attack, but that requires a reference of apersonal and usually insulting nature, and Parent 2 commits no such cheap shot.

• Words like “obviously” and “clearly” are Conclusion Keywords just as certainly as“therefore” and “thus.” Use them, whenever they appear, to pinpoint the conclusionearly in your attack on the stimulus.

2. (C)We seek an additional piece of evidence that, when combined with what’s alreadyprovided, will confirm that “no ski resort owners are lawyers.” We can ignore the thirdstatement, as well as choices (A) and (B), because “some” statements get us no closer toinferring what must be true about “all” or “none” of a group. Next, we want to force adistinction between ski owners and lawyers, but with statement 3 removed no evidenceabout lawyers remains. And of course we need some. (C) is the only remaining choice thatmentions lawyers, so it must be correct.

Even if you didn’t take this intuitive approach, you could still reason it out directly. If allcattle ranchers dislike long winters while all ski resort owners like them (never mind thereason provided by statement 2, that’s just filler), then those two groups have no membersin common. And if, as (C) says, all lawyers fall within the group of cattle ranchers, thenlawyers and ski resort owners can have nothing in common either, and that’s theconclusion we were told to reach.

• LR sections often start off with low difficulty questions. But don’t be dismayed orembarrassed if an early question is of a type (like formal logic) that you may not becomfortable with. You can always come back to it later.

• A formal logic reminder: Usually, not every LSAT stimulus sentence is needed forthe process of drawing a formal conclusion.

Page 25: 18-PrepTest 18 Explsdl.keywin.org/4/9/49d31fb5129d388a6dd62829ca482add.pdf · 2012. 4. 13. · wrong choices in a “could be false” question are statements that must be true.)

LSAT PREP _____________________________________________________________ LSAT Test XVIII Explained: Section II

© K A P L A N 23

3. (A)The question and the Citizen focus on the same issue: the allegation that it’s possible toalter the city council’s composition in a major way. How so? By campaigning ferociouslyagainst the many incumbents on the current council. Oh, of course, the Citizen makes oneexception: She’s going to vote for her incumbent, who’s doing a good job. But if everyonein Mooresville did likewise—campaign against incumbents generally but vote for theirown incumbent—then no incumbent would lose! To effect the magnitude of change theCitizen desires, then, it has to be true that enough Mooresvillians vote for non-incumbentsto throw at least some of the rascals out. That’s (A), in so many words.

(B) The number of total voters has no effect on the composition of the council. The issue iswhom those voters, however many there are, select.

(C) and (E) are likewise outside the scope. Candidates’ previous electoral history (C) andability to serve well (E) have nothing to do with what the Citizen is arguing about, i.e. thepotential for council membership change through the ballot box.

(D) goes too far. A “substantial” change is what the Citizen seeks, and that could beeffected even if, contrary to (D), some of the council seats are being contested by non-incumbents.

• As mentioned during Question 2, while many LR sections do start out relativelyeasy, LSAC usually throws in a lengthy or unusual question like this one. They wantto determine which test takers will get mired in it and which will pass through oraround it. Don’t fall for that gambit. See to it that you never get bogged down,especially early on in a section.

4. (D)David may perhaps be forgiven for misunderstanding Marianna’s point, because shejumps around to three different topics: the partial success of drunk drivingcountermeasures; the need for additional measures; and the likelihood of drunk driverscausing harm. But you, of course, need to follow her topics carefully, and recognize thatDavid is speaking to a different topic altogether, namely, the likely harm that the drunkdriver may self-inflict. Marianna is concerned about death or injury in accidents in general,and David, guilty of shifting the scope, has missed that.

(A) No he doesn’t. A self-contradiction is when one utters two statements that can’t both betrue. But everything David says can be simultaneously true. He contradicts or takes issuewith Marianna, yes, but that’s another thing altogether.

(B) This is a wordier definition of what’s known as “circular reasoning,” in which oneassumes one’s own conclusion—in other words, one’s evidence and conclusion arebasically identical. But that’s not David’s misstep: His conclusion (that Marianna isoverstating things) is indeed backed up by separate evidence (his second sentence).

(C) As just noted under (B), David does provide evidence. It’s just not to the point she’smaking.

Page 26: 18-PrepTest 18 Explsdl.keywin.org/4/9/49d31fb5129d388a6dd62829ca482add.pdf · 2012. 4. 13. · wrong choices in a “could be false” question are statements that must be true.)

LSAT PREP _____________________________________________________________ LSAT Test XVIII Explained: Section II

24 © K A P L A N

(E) See Q. 1(E), above, on the requirements for an ad hominem argument. To say “I disagreewith you,” as both Parent 2 and David do, is not in and of itself a personal attack.

• Always read each LR question stem first, and you’ll benefit from the clues theyprovide. You don’t have to figure out that David commits a reasoning error, you’retold it. Get in the habit of deciphering the question stem—and stay in that habit!

• Always be attuned and on the lookout for scope shifts, especially when you knowgoing into the stimulus that something’s logically awry.

5. (D)To “appeal to authority” is to gain support for your argument by looking to someonewhom you believe to be an expert: “This must be a good recipe, because PaulPrud’homme recommends it.” No such authority is cited here: The views of Hitler andStalin aren’t cited as backup. In fact, the two tyrants are used as extreme, emotion-ladenexamples (A) to illustrate how the normally positive virtue of self-confidence can lead toarrogance (C), the implication being that Hitler and Stalin are good examples of thephenomenon (E) and that arrogance is in fact a bad thing, a “vice” (B).

• Questions 4 and 5 provide an excellent opportunity for gauging whether you’recomfortable with the abstract language typical of “method of argument” questions.If you got either one wrong, or even came close to getting it wrong, figuring out whyyou were tempted to a wrong answer and away from a credited one may tell you a lotabout how you need to improve your handle on this question type.

6. (B)The easy-to-locate conclusion, that oil rigs don’t hurt sea-bottom animals, is based on theevidence that there were no differences between the animal communities near rigs andthose “several miles from” rigs. That reasoning, in turn, depends on the assumption thatthe only impact oil rigs have on sea-bottom animals is in their immediate vicinity. But if, as(B) says, discharged surface rig oil often travels a long way before it impacts the seabottom, then that assumption is untrue, and the link between evidence and conclusion issevered. Now, the conclusion is premature until someone examines the effect of oil rigs onanimals within the radius of the traveling oil.

(A) Too hypothetical to be damaging. We don’t know whether such a drop in fish catcheshas in fact occurred and, if it has, whether in fact it was due to oil rigs.

(C)’s redefinition of the harm that “sewage and industrial effluent” could cause at the seabottom is nothing more than an irrelevant distinction. Even if this is true, the link betweenevidence (no significant differences) and conclusion (no adverse effect) isn’t damaged.

(D) Oil discharge that never makes it to the sea bottom is outside the scope of thisinvestigation—see sentence 1 of the stimulus.

Page 27: 18-PrepTest 18 Explsdl.keywin.org/4/9/49d31fb5129d388a6dd62829ca482add.pdf · 2012. 4. 13. · wrong choices in a “could be false” question are statements that must be true.)

LSAT PREP _____________________________________________________________ LSAT Test XVIII Explained: Section II

© K A P L A N 25

(E) We have no idea whether oil rigs are more or less likely to be built on soft vs. rockyground, so (E), like (C), poses an irrelevant distinction. And so what if rigs are built on softground? If the stimulus evidence is correct, that persistent contaminating oil has no effecton the animals anyhow.

• This question beautifully demonstrates that weakening an argument doesn’tnecessarily mean disproving the conclusion; rather, it means severing the linkbetween evidence and conclusion. If (B) is true, the researchers could still becorrect—it still could be true that oil rigs in fact do no harm to sea bottom animals.But one could not so conclude on the basis of the given evidence, and that’s the point.The researchers would have to come up with new or additional evidence to drawtheir conclusion. Hence (B) has weakened it.

• Question 6 also illustrates the value of scanning the stimulus for a clue as to theconclusion. “The researchers concluded that” is a great place to begin your analysis.

7. (B)The author begins with a common description of scientists: They assume that somethingisn’t true until there’s proof that it is true. He shows that this attitude would lead to acontradiction. If a scientist doesn’t know whether a food additive is safe, he or she mustassume that it’s not safe, because it hasn’t been proved safe. On the other hand, since theadditive hasn’t been proved UNsafe, the scientist must assume that it’s safe. So thecharacterization of scientists would require that scientists hold contradictory beliefs;therefore, the author says, the characterization can’t be accurate.

(A) The author never discusses the motives of those who formulated the general statementabout scientists; in fact, we have no idea who those people are.

(C) The author shows that the methodology described by the first statement could lead toholding contradictory beliefs; however, the author never takes an independent secondstatement as true.

(D) and (E) can both be ruled out by the inclusion of the word “uninformative.” This wordis way too weak to describe a statement the author dubs “clearly wrong.” If you missedthat, note that the author never compares the numbers of cases when the statement is true tothe number of cases when it’s false (D), and we can’t be clear on what is meant by“independently testable” in (E).

• Make sure you isolate the conclusion. The conclusion here is not that the statementleads to a logical contradiction, but that the statement can’t be an accurate picture ofhow scientists think. If you didn’t see that, you might have been thrown by thephrase “implausible consequences” and been drawn to (C), because it speaks of acontradiction.

• Pay attention to the strength of an author’s convictions, as denoted by the tone orspecific phrases used. Concluding that a statement is “clearly wrong” is muchstronger than saying that it is merely “uninformative,” and that alone allows us tokill (D) and (E).

Page 28: 18-PrepTest 18 Explsdl.keywin.org/4/9/49d31fb5129d388a6dd62829ca482add.pdf · 2012. 4. 13. · wrong choices in a “could be false” question are statements that must be true.)

LSAT PREP _____________________________________________________________ LSAT Test XVIII Explained: Section II

26 © K A P L A N

8. (E)The author acknowledges that most of the homicides in question are unpremeditatedfamily assaults, but says that even if such assaults are increasing, they probably wouldn’tresult in deaths if it weren’t for all the lethal knives floating around. (E) gives an effectivetwo-part criticism of the author’s knife-obsession. If the knives she’s talking about areregular household knives, such knives have always been around, and something that hasalways been the case can’t be blamed for a change such as the rise in homicides. On theother hand, if the knives are actual weapons, designed for combat, they are not generallyavailable, as the author claims—again, they can’t be responsible for the increase inhousehold homicides. Either way, the rise in homicides can’t be blamed on the availabilityof knives.

(A)’s outside the scope, and entirely misses the author’s point: Her argument is based onher belief that most homicides are the result of unpremeditated assaults, while this choicefocuses on people intending to cause death.

(B) The author doesn’t base her argument on the fact that unpremeditated family assaultsare increasing—she merely tries to show that her argument would work even if suchassaults were increasing. Whatever the number of assaults, she argues, knives are to blamefor the deaths.

(C) Outside the scope: Clearly, the fact that knives are used in non-family homicides aswell as family homicides doesn’t weaken the author’s claim that the prevalance of kniveshas caused an increase in Britain’s homicide rate.

(D) On the contrary, the author specifically says that most homicide deaths occur as aresult of unpremeditated family assaults.

• When the author is claiming to find an explanation for an increase (or decrease) insomething, make sure the explanation actually accounts for the change, not just thefinal state.

• Watch out for those blatant scope shifts: The author clearly focuses onunpremeditated homicides, while two choices, (A) and (D), insist on discussing theopposite. Dispose of them quickly.

9. (E)A vitamin is a vitamin is a vitamin, says the author; there’s no difference between man-made vitamins and naturally occurring vitamins. So why pay more for vitamins madefrom supposedly higher quality ingredients? Did you catch the scope shift there? Thevitamin component may be the same, but who’s to say that all the ingredients are the same?Maybe there are higher quality non-vitamin ingredients (natural ingredients that makevitamins easier to absorb into the bloodstream, additional dietary supplements, minerals,whatever) in the “higher-quality” or “more natural” pills that make them nutritionallysuperior and worth paying more for.

Page 29: 18-PrepTest 18 Explsdl.keywin.org/4/9/49d31fb5129d388a6dd62829ca482add.pdf · 2012. 4. 13. · wrong choices in a “could be false” question are statements that must be true.)

LSAT PREP _____________________________________________________________ LSAT Test XVIII Explained: Section II

© K A P L A N 27

(A) Whoa Nellie . . . The author doesn’t assume that all vitamin pills are worthless; herpoint is that there’s no reason to choose one type of vitamin pill over another based on theiringredients.

(B) The author doesn’t assume that natural-ingredient vitamins always cost more; shesimply argues that when they do cost more, there’s no reason to buy them.

(C) is absurd; the author is contrasting synthetic vitamins with natural vitamins and needn’tassume that all vitamin pills contain synthetic vitamins.

(D) A producer may honestly believe that their brand is superior, and therefore wouldn’tbe guiltly of false advertising. It’s the author who feels there’s no difference betweenvitamins, but we can’t infer that the author thinks the producers agree and are thereforeguilty of purposefully misleading the public.

• Remember that for assumption questions, you need something the argumentrequires, not just something that may be consistent in some way with the author’spoint, like (D). It’s possible that some producers are guilty of false advertising, butthe logic of the argument is not dependent upon this.

10. (E)The author begins by telling us how “most people” feel; they’re insulted by the suggestionthat they may not understand what they really want. The rest of the stimulus is devoted toexplaining why these people are misguided to feel this way—self-knowledge is hard tocome by, it’s easier just to go with the flow and accept what society says, etc. So the point ofthe argument is to show that it’s true that people aren’t necessarily the best authorities as towhat their real wants are, despite the fact that most people don’t want to hear it.

(A) The author does say acquiring self-knowledge can be risky, but that’s not her mainpoint; she makes that point in order to show why many people never bother to acquire self-knowledge.

(B) The author doesn’t argue that self-knowledge is undesirable, only that it’s not as easy toacquire or as common as people think.

(C) is an overstatement and a distortion; the author says that many people don’t know whatthey do want, not that all people don’t want what they should want.

(D) is far too vague. The author never speaks of “difficult decisions” in general; her point islimited to the fact that people often avoid doing the work necessary to gain self-knowledge.

• When a stimulus begins by telling you what “most people” think, or what “acceptedtheory” holds, or what “used to be believed,” chances are good that the purpose ofthe stimulus is to attack that belief.

• A small point, but notice that this is the third (E) answer in a row. There’s no rhymeor reason to the sequence of correct answers, so don’t look for a pattern or expect thecorrect answers to be spread evenly.

Page 30: 18-PrepTest 18 Explsdl.keywin.org/4/9/49d31fb5129d388a6dd62829ca482add.pdf · 2012. 4. 13. · wrong choices in a “could be false” question are statements that must be true.)

LSAT PREP _____________________________________________________________ LSAT Test XVIII Explained: Section II

28 © K A P L A N

11. (B)The paradox is simple: Pesticide use has increased, but crop loss to insects has alsoincreased, despite the fact that virtually the same amount of acreage is being planted. Wewant a choice that will help explain the ineffectiveness of the pesticides in reducing insectdamage to crops. (B) does the job by introducing two new factors that help explain theincreasing crop loss to insects. First, pesticide-resistant strains of insects have developed;thus, obviously, pesticides are less effective in reducing the insect population. Second,farmers have been abandoning crop-rotation, which disrupted the insects’ food supply andpresumably made it more difficult for them to thrive. With a steady food supply andincreased resistance, it’s easy to see how the surprising result came about.

(A), (D) The actions of a group of the extension agents or farmers described in thesechoices in no way change the overall fact that pesticide use has increased tenfold. Neitherchoice provides any means of resolving the discrepancy.

(C) That there’s been a shift to a new type of pesticide that is just as effective against insectsis of no help in explaining the increasing damage caused by insects.

(E) speaks of the effect of pesticides on consumers, and their reaction to it. We’re onlyinterested in the effect (or lack thereof) of pesticides on insects.

• You’re being asked to explain a paradoxical situation; that means the correct answermust deal with the facts of the situation. (A) and (D) represent a common wronganswer-type: They both focus on subsets of the whole situation and give extraneousinformation that doesn’t in fact explain anything. We could think of one hundredother cases involving specific groups that advocated a particular position orfollowed a certain action plan regarding pesticide use, without substantively alteringthe hard and fast “fact” that “pesticide use in the United States has increasedtenfold.” Beware of these choices that contain such nonsense—they resolve nothing.

12. (D)The author cites a claim made by politicians, takes exception to the claim (“this claim isplainly false”), and then offers an example in an attempt to demonstrate why the claim isfalse. This analysis alone should lead you towards (D); the author certainly attempts torefute the politicians’ claim by furnishing an example. If we look deeper, we find that therest of (D) pans out as well: the fact that divergent living standards in regions withincountries have nevertheless allowed for stable economies is clearly intended to imply thatsuch divergent living standards in Europe as a whole wouldn’t hinder a stable economyshould monetary union occur there. In other words, the author argues by bringing up whatshe considers to be a similar case where these factors coexist.

(A) The author takes no account of a time element in making her counter-argument.

(B) No; this would mean that the author cited a case where monetary union amongEuropean nations occurred in the past without leading to economic chaos. This simplydoesn’t happen.

Page 31: 18-PrepTest 18 Explsdl.keywin.org/4/9/49d31fb5129d388a6dd62829ca482add.pdf · 2012. 4. 13. · wrong choices in a “could be false” question are statements that must be true.)

LSAT PREP _____________________________________________________________ LSAT Test XVIII Explained: Section II

© K A P L A N 29

(C)’s way off. The author argues, by way of example, that the feared consequence(economic chaos) won’t occur in Europe, even with divergent living standards in thecountries concerned.

(E) Huh? What recommendation? What’s this wishy-washy “neither true nor false” stuff?The author makes it fairly clear what she thinks about this claim, just as we’ll make it fairlyclear what we think about this choice: It stinks.

• Always read the question stem first in Logical Reasoning. You can see clearly fromthis stem that this is a Method of Argument question, which immediately tells you topay close attention to the general structure of the argument.

• Notice the phrase “assumed to be relevantly similar” in the correct choice. Don’tworry about the fact that the situations may not be similar; the author is in factassuming that monetary union exists among the regions cited. But we’re not askedfor the assumption—we’re asked for the argumentative method, which just sohappens to include the assumption on the part of the author that the situation sheinvokes is indeed similar to the one she attempts to refute.

13. (C)The conclusion, signaled by “hence,” is that a single course of action has had an impact on“everyone.” Of all the choices, only (C) fits that description, so it must be correct.

(A)’s conclusion characterizes, or ascribes a value judgment to, a single course of action(the surgery wasn’t heroic), and that in and of itself departs from the stimulus. Worse, thatcharacterization is based on a hypothetical principle (“If an act is not heroic...”) rather thanthe concrete principle that the simulus uses (sentence 2).

(B) and (E) do nothing more than reason from, and to, general principles. Each lacks a keypoint of a reference to a single course of action, like the dean’s decision (stimulus) orIsabel’s rescue (C).

(D) is a little more specific since it focuses on “fire fighters,” but (D), like (B) and (E), failsto highlight a single course of action in its conclusion. And (D)’s “are often expected to”has no counterpart in the stimulus.

• Comparing the nature of the stimulus conclusion to that of the conclusion in eachchoice doesn’t always lead, as it does here, to an instant right answer. Look ahead, forinstance, to Question 17 in this section, where five of the six conclusions aresuperficially similar; there, we’ll have to delve deeper. However, this tactic is oftenenough of a timesaver that it’s worth practicing.

Page 32: 18-PrepTest 18 Explsdl.keywin.org/4/9/49d31fb5129d388a6dd62829ca482add.pdf · 2012. 4. 13. · wrong choices in a “could be false” question are statements that must be true.)

LSAT PREP _____________________________________________________________ LSAT Test XVIII Explained: Section II

30 © K A P L A N

• Practice extrapolating from the text in every Parallel Reasoning example youencounter. Did you notice the elements highlighted above? The stimulus and correctchoice contain a reference to a single specific course of action; seeing that, you caneliminate some choices right off the bat. Another choice contains a hypotheticalelement (“if an act is not heroic...”), an aspect that doesn’t appear in the original. Takenote of the elements that characterize the original, search for them in each of thechoices, and cross off choices that contain elements that don’t match.

14. (D)The professor differentiates between “communication” and “language”; not allcommunication is language, he says. The human communication system is definitelylanguage; in fact, if we didn’t use language, we wouldn’t qualify as human. The studentaccepts this distinction, but “objects” that the professor hasn’t explained how he knows thatother animals don’t also have communication systems that qualify as languages. Plainly,the student has taken the professor to say (D), that only humans have languages. Theprofessor contends that “if we didn’t use language, we wouldn’t be human.” Unbeknownstto the confused student, this is not the same as saying that “if a species isn’t human, itdoesn’t use language.”

(A) First, (A) is too vague; it doesn’t even mention the idea of “language” or“communication.” Second, the student asks how the professor can be sure that species don’tshare a particular trait (language), so she can’t believe that he said that species do sharetraits.

(B) Outside the scope: Neither the professor nor the student says anything about traitsother than languages, so the student evidently doesn’t believe that the professor wasreferring to such traits.

(C) Both the student and the professor speak of languages as a subset of communicationsystems, with the student suggesting that some animal communication systems mayqualify as languages. Neither speaker refers to language as existing withoutcommunication.

(E) Since the student doesn’t say anything about communication between species, shedoesn’t believe that the professor was speaking about that.

• Did you recognize the student’s mistake as a common logical error? In effect, theprofessor says “the language trait is necessary to determine humanity.” The studenttakes this to mean “the language trait is sufficient to determine humanity,” andresponds accordingly.

• For a choice to be a plausible misinterpretation (or a plausible “point at issue”), itmust deal with a topic that is referred to by both speakers. That’s why it’s soimportant to focus on topic and scope in dialogue questions that ask you, in someway, shape, or form, how the two arguments relate.

Page 33: 18-PrepTest 18 Explsdl.keywin.org/4/9/49d31fb5129d388a6dd62829ca482add.pdf · 2012. 4. 13. · wrong choices in a “could be false” question are statements that must be true.)

LSAT PREP _____________________________________________________________ LSAT Test XVIII Explained: Section II

© K A P L A N 31

• On a misinterpretation question, it’s important to get the tone of the secondspeaker’s reply. The student says “how do you know that X aren’t languages,” so shemust believe that the professor has committed himself to the position that X aren’tlanguages.

15. (A)The crux of the argument is that the environmentalist decides that oil industry safetystandards are the responsibility of the national government; the only reason he gives forthis is the fact that the oil industry won’t clean up its own act. He’s assuming that these arethe only two choices: the oil industry impose stricter standards or the national governmentimpose stricter standards. That’s the assumption (A) points out; the environmentalistassumes that no other agency, such as a state or local government, could regulate industrysafety standards.

(B) The environmentalist never takes costs into consideration—his concern is promotingsafety, not saving money, so he needn’t assume anything about costs.

(C) Although the environmentalist says double-hulls should be put on tankers, there’s noimplication that this must involve repairing or replacing the industry’s “aging fleet oftankers,” so (C) isn’t assumed.

(D) The environmentalist certainly doesn’t assume that regulations are the result of aprocess of negotiation; on the contrary, he gives the impression that regulations must beimposed on an unwilling oil industry.

(E) Although the environmentalist believes that stricter safety standards are necessary forthe oil industry, he needn’t assume that environmental concerns outweigh all financialconcerns whenever safety standards are developed; that’s much too broad.

• Pay attention to unusual turns of phrase in the stimulus; on the LSAT language isused precisely, and if something is phrased a little oddly, there’s a good reason.When the environmentalist says “it is the national government that must... “ he’ssaying “only the national government can...” and that’s the key to the question.

• Preview the question stems! The stem for 15 tells you that you can (and should)answer the first question of this pair without even looking at what the industryrepresentative says. Why run the risk of confusing the two arguments when the stemtells you that you can get this point from the first argment alone?

• Don’t be roped into extreme sounding answer choices like (E). Is this drastic notionreally necessary in order for the environmentalist’s argument to stand? Clearly not.

Page 34: 18-PrepTest 18 Explsdl.keywin.org/4/9/49d31fb5129d388a6dd62829ca482add.pdf · 2012. 4. 13. · wrong choices in a “could be false” question are statements that must be true.)

LSAT PREP _____________________________________________________________ LSAT Test XVIII Explained: Section II

32 © K A P L A N

16. (D)The industry representative argues that the industry alone should be responsible fordevising safety standards, because only the industry fully understands the factorsinvolved. He uses the environmentalist’s own example against him: The double-hullfeature, which the environmentalist thinks is a great idea, would raise new safety issuesand would cost a great deal for both industry and consumers. The representative is usingthe example to show that outsiders, however well-meaning they might be, really don’tknow what they’re talking about when it comes to the oil industry. (D) supports therepresentative’s argument by confirming his claim that double-hulls raise important safetyissues. If double-hulls increase the risk of explosion, then maybe they’re not such a goodidea, and maybe it is dangerous to let non-industry people try to regulate the oil industry.

(A) Au-contraire—this is a case where the double-hull performed well, which supports theenvironmentalist’s position.

(B) describes a specific way in which government regulation would operate, but it doesn’tpertain to the industry representative’s argument. Nothing in (B) confirms therepresentative’s claims about the drawbacks of government regulation.

(C) does suggest that over time the double hulls may cost the industry some money, byforcing old ships to be updated or retired, but it doesn’t say how much money, and it givesno reason to think the costs will become “burdensome” to consumers. Also, it deals with“proposed” legislation. Who even knows if this legislation will ever come to pass?

(E) Like (B), (E) describes how government regulation operates, without specificallyshowing that this confirms the representative’s claims that government regulation will bemisguided and burdensome.

• An answer choice can strengthen an argument by directly supporting its conclusion,by strengthening the connection between its conclusion and its evidence, or byconfirming (or adding to) its evidence. The correct answer here took the latter road,by confirming the claim about the dangers of double-hulls. You may have beenexpecting a choice that more directly tackled the idea of government regulation, butkeep in mind that shoring up evidence is one valid way to strengthen an argument.

17. (C)The conclusion—that a particular person’s belief is justified—is more or less parallelled byfour choices but at least allows us to throw out (D) right away. (D) is about a belief held bymany people, not one person, and the phrase “both explained and justified” has no parallelin the stimulus.

Delving deeper, we see that according to the author, Arnold’s paranoia about his co-workers seems irrational but actually is valid. And why? Because it’s based on experiencesin which someone put Arnold down when he was young. (Huh?)

Page 35: 18-PrepTest 18 Explsdl.keywin.org/4/9/49d31fb5129d388a6dd62829ca482add.pdf · 2012. 4. 13. · wrong choices in a “could be false” question are statements that must be true.)

LSAT PREP _____________________________________________________________ LSAT Test XVIII Explained: Section II

© K A P L A N 33

This author sounds as loony as Arnold (the fact that Arnold’s sister demeaned hisambitions and achievements is hardly “clear justification” that Arnold’s officemates arebeing two-faced), but it’s no loonier than the author and subject of (C): Here, Joan’s badchildhood experiences with her father (brought on by playing with the cat) are bafflinglycited as conclusive proof that her current belief that cats make bad pets is justified. In boththe original and choice (C), the flawed logic breaks down to this: if a particular childhoodexperience helps to explain why someone holds a certain belief, then that belief must bejustified.

(A)’s parts are general while those of the stimulus, and (C), are particular. Sheldon’s beliefstems generally from his childhood; the belief discussed in the stimulus is a current beliefthat stems from an identifiable and particular childhood syndrome. And Sheldon’sconclusion is a general principle about how to succeed in business without really trying,while Arnold’s “stubborn belief” has to do with one particular set of colleagues in oneparticular context.

(B) does present a specific early influence—Emily’s choking fit and her belief that applesare bad for you—but brings in pesticides out of nowhere. The pesticides (which may ormay not have been present in Emily’s single apple) are part of new evidence that seems togo hand in hand with Emily’s anti-apple bias, but the stimulus contains no such newevidence.

(D) See above.

(E) would be closer to the stimulus if Sumayia were drawing a conclusion aboutAnthony’s lack of drive from other pianists in her early experience who likewise lacked drive. Asit is, however, (E) simply supports Sumaiya’s opinion about Anthony by arguing that sheknows pianists generally. That she must be right about Anthony is probably anoverstatement, but it’s nowhere near as faulty an argument as the stimulus, or (C) for thatmatter.

• Don’t “nickel-and-dime” Parallel Reasoning stimuli and choices. Read broadly forthe kind of evidence used and the kind of conclusion drawn.

18. (A)The author praises the documentary for looking beyond the customary “save-the-wildlife”cliches and examining the situation of the elephant in East Africa in greater depth, showingthe negative effects that elephants have on the people of the country. In a region pressed forfood, the elephant turns out to be a voracious, uncontrollable pest. She ends by saying thatthe example of the elephant in East Africa illustrates something, and we’re to guess whatthis something is. Since the whole paragraph describes the dangers posed to humanwelfare by elephants, the author is most likely to say (A), that the example of the elephantin East Africa illustrates that the preservation of wildlife may pose dangers to humanwelfare.

Page 36: 18-PrepTest 18 Explsdl.keywin.org/4/9/49d31fb5129d388a6dd62829ca482add.pdf · 2012. 4. 13. · wrong choices in a “could be false” question are statements that must be true.)

LSAT PREP _____________________________________________________________ LSAT Test XVIII Explained: Section II

34 © K A P L A N

(B) takes off in the wrong direction. The author is interested in the situation in East Africa,as presented in the documentary, because it shows a conflict between animal and humanwelfare; nothing in the paragraph indicates an interest in the situation of elephants ingeneral.

(C) contradicts the paragraph’s claim that the documentary went beyond the usual pieties,so it’s not a logical completion.

(D) The author hasn’t said anything about how elephants should be controlled, or about“agricultural agents” or “wildlife conservationists,” so (D) doesn’t fit; there are simply toomany new ideas introduced.

(E) is wild; the paragraph can’t draw sweeping conclusion about food shortages “in anycountry” on the basis of a documentary about East African elephants.

• On fill-in questions you need to understand how the blank fits in the sentence andhow the sentence fits in the paragraph. Here the final sentence is a summing-up; therest of the paragraph is supposed to provide a specific case study that supports ageneral claim in the last sentence.

19. (B)This isn’t as complicated as it sounds. A rain cloud contains a lower proportion of oxygen-18, as compared to the more common normal oxygen. In rainfall, however, proportionallymore oxygen-18 falls than normal oxygen. In other words, a rain cloud seems to be usingup oxygen-18 at a greater rate than it uses up normal oxygen; you’d therefore expect theproportion of oxygen-18 in a cloud to become steadily less as the cloud releases rain.However, scientists who measured the oxygen content in clouds going from the AtlanticOcean over Amazon rain forests found that the oxygen-18 content in the clouds remainedconstant. (B) best resolves the discrepancy by explaining that the progress of the cloud overthe Amazon is not simply a matter of losing water (and oxygen) in rain, but also of gainingwater (and oxygen) from the rain forests. If the cloud is being continually replenished, it’spossible (and even likely) that it gains oxygen-18 as fast as it loses it.

(A) makes an irrelevant comparison between tropical forests and unforested regions; itnever addresses the issue of why, when clouds were observed passing over the Amazonforest, the expected change in oxygen content didn’t occur.

(C) says that when rain clouds pass over the Amazon forests they shed all the water theycollect from the Atlantic Ocean, but this doesn’t explain why the proportion of oxygen-18in the clouds doesn’t change as the clouds drop rain.

(D) is way beyond the scope. A comparison of the rain lost in “river runoffs” to the rainrecycled back into the atmosphere tells us nothing about the oxygen content of rain clouds.

(E) is irrelevant; we want to know why the oxygen-18 content of rain clouds doesn’t changeover the Amazon forests, and the fact that oxygen-18 is a poor indicator of the forests’general atmospheric effect is of absolutely no use.

Page 37: 18-PrepTest 18 Explsdl.keywin.org/4/9/49d31fb5129d388a6dd62829ca482add.pdf · 2012. 4. 13. · wrong choices in a “could be false” question are statements that must be true.)

LSAT PREP _____________________________________________________________ LSAT Test XVIII Explained: Section II

© K A P L A N 35

• It’s hard for us to recommend that you skip a double-question stimulus, becauseyou get two questions for the price of one. It’s usually worth taking the extra time totry to understand the passage. However, if you find that you just can’t grasp what’sgoing on, and you’re just wasting time, then you have no choice; come back and lookat it later, hopefully with a clearer head.

• Try to strip such dense stimuli down to the barest essentials: “Something (oxygen-18content) should change when rain clouds pass over tropical forests, but it doesn’tchange.” Even if that’s all you take away from the stimulus, (B) is the only choice thataddresses the lack of change in the clouds.

20. (A)The stimulus says that in a rain cloud, water molecules containing oxygen-18 are rarer thanwater molecules containing normal oxygen. Since rain clouds are entirely made up ofwater molecules, that means that rain clouds contain more regular oxygen than they dooxygen-18. Therefore, we can infer (A): the rain clouds formed over the Atlantic containmore regular oxygen than oxygen-18.

(B) flatly contradicts the stimulus, which says that the oxygen-18 content of clouds passingover the Amazon forests remains fairly constant.

(C) is a misinterpretation. The stimulus is very careful to say that the cloud loses a higherproportion of its oxygen-18 to rain than it loses of its regular oxygen; for example, it maylose 1/2 of its oxygen-18 water molecules and only 1/3 of its regular-oxygen watermolecules. But since there are more regular oxygen water molecules in the cloud to beginwith, it doesn’t follow that rain water necessarily contains more oxygen-18 than ordinaryoxygen.

(D) contradicts the stimulus, which says, as we just saw, that the cloud loses a greaterproportion of oxygen-18 to rain than it loses of oxygen.

(E) makes a false comparison; the stimulus says nothing on which to base a comparisonbetween the amount of oxygen (of either type) that the cloud loses and the amount thatremains.

21. (A)It’s very difficult, says the author, to determine the artist responsible for a painting that’s twoor three hundred years old; as a result, people are likely to put great weight on thetraditional attribution, since at least that has the advantage of “historical continuity”—that is,it’s been handed down from a time much closer to when the painting was originally done.Therefore, historians arguing against the traditional attribution are unlikely to persuademany people unless they can come up with a convincing alternative. (A) is the best reasonamong the bunch why people shouldn’t put such credence in traditional attributions byshowing that there’s a chance that any given traditional attribution was inspired by a desirefor illicit gain (and that the continued acceptance of that attribution was also inspired by artdealers’ desire for gain). If (A) is true, the fact that an attribution has been traditionallyaccepted isn’t necessarily a compelling reason to think that it’s accurate.

Page 38: 18-PrepTest 18 Explsdl.keywin.org/4/9/49d31fb5129d388a6dd62829ca482add.pdf · 2012. 4. 13. · wrong choices in a “could be false” question are statements that must be true.)

LSAT PREP _____________________________________________________________ LSAT Test XVIII Explained: Section II

36 © K A P L A N

(B) is irrelevant, since we’re not concerned with questions of attribution at the time of awork’s creation, but with the validity of a traditional attribution two or three hundredyears later.

(C) echoes the stimulus by implying that it can be difficult to figure out who is responsiblefor a painting, but gives no reason to mistrust the traditional attribution, and provides nobetter way of coming up with an attribution.

(D) refers to the effect that an attribution can have on people’s perception of a painting, butsays nothing about how an attribution is arrived at in the first place, which is what we’reinterested in.

(E) is simply a matter of definition, explaining that “attributing” a painting to a masterdoesn’t mean that he painted every last line. (E) doesn’t, however, say anything aboutwhether a traditional attribution is apt to be reliable.

• You need not perfectly understand every term in the stimulus to use the passage’sclues to get the right answer. You’re asked about “traditional attribution,” so focuson the second sentence where it’s defined. There you see traditional attribution isvalued for “historical continuity.” Even if you don’t know what this means exactly, itclearly refers to something happening over a long period of time, and (A)’s point thatpeople have always had motives to misattribute paintings should strike you.

• Even if some terms are ambiguous, you should still be able to pre-phrase what it isyou’re looking for. Basically, we’re looking for something that shows that“traditional attribution” of disputed paintings may not be so great (which is just aneasier way of saying “should not have special weight”).

22. (B)Many of the results of early scientific work have been shown to be false. Scientists areinterested above all in discovering the truth. Yet it’s valuable for today’s scientists to studyaccounts of earlier scientific work. We’re supposed to reconcile these statements, andchances are you could have pre-phrased the answer to this one: Anything that shows someother benefit (that is, besides discovering the truth) from studying the earlier accounts willdo the job. (B) does the trick: By studying earlier scientific work, even work whose resultslater proved incorrect, scientists can learn valuable lessons in methodology.

(A) What does (A) mean? The accounts are mistaken but aren’t known to be? In that case,why study inaccurate accounts? Or does (A) mean that the accounts “aren’t known to bemistaken” in the sense that there’s no reason to think that they’re mistaken? If so, (A) stilldoesn’t explain why today’s scientists should study accurate accounts of flawed earlierwork.

(C) speaks of the contribution that scientists can make to the science of their time withflawed work; we need to know what contribution such work can make to scientists of alater time in order to resolve this seeming paradox.

Page 39: 18-PrepTest 18 Explsdl.keywin.org/4/9/49d31fb5129d388a6dd62829ca482add.pdf · 2012. 4. 13. · wrong choices in a “could be false” question are statements that must be true.)

LSAT PREP _____________________________________________________________ LSAT Test XVIII Explained: Section II

© K A P L A N 37

(D) We don’t care about this—we’re interested in explaining why, despite evidence whichseems to lean to the contrary, these scientists should be thoroughly familiar with earlierscientific work.

(E) does the opposite of what we want: We’re looking for a reason why it’s valuable tostudy earlier, albeit flawed, work. (E) tells us that some of the better research of today doesjust fine without bothering to address earlier work.

• You’ll often run into an easy question or two at the end of the section, which is whyit’s so important to try to get a decent look at all the questions. As you know, thismeans not getting bogged down on early tough questions, but leaving them andcoming back to them if you have time. This should make perfect sense—if you haveto venture a guess on a question, why not at least make it a question that youdetermine to be killer? You may very well get such a question wrong even if youspent three minutes on it. The double whammy is that you would also probablysacrifice an easy question or two towards the end.

• Pay special attention to the existence of alternative explanations when searching foranswers to Paradox questions.

23. (E)Ah, formal logic. We’re looking for the one choice that could not be true; that means wewant a choice that flatly contradicts something in the stimulus. What does the stimulus tellus about teachers who enable their students to make their own decisions? Teachers can onlyenable students to make their own decisions when they have the power to make decisions intheir own classrooms. Therefore, (E) is impossible; it can’t be that teachers who have enabledstudents to make their own decisions do not have the power to make decisions in their ownclassroom.

(A) According to the third sentence, students’ capability to make their own decisions isessential to their becoming independent learners; however, that capability does notguarantee that they will become independent learners (i.e., it’s necessary, but not sufficient).Hence, it’s possible that teachers have enabled their students to make their own decisions,yet those students haven’t become independent learners.

(B), (C) Either of these is quite possible; these teachers may or may not be effective. Even iftheir students have become successful learners (which, we know from (A) need not be thecase), this doesn’t mean these teachers must be effective. Sure, it’s possible, but we can’t tellfor sure. All we can deduce from the given in the question stem is that these teachers mustbe able to make decisions in their classroom, which is why correct choice (E) is false, andincidentally, explains why (D) not only could be true, it in fact must be true.

(D) See above.

Page 40: 18-PrepTest 18 Explsdl.keywin.org/4/9/49d31fb5129d388a6dd62829ca482add.pdf · 2012. 4. 13. · wrong choices in a “could be false” question are statements that must be true.)

LSAT PREP _____________________________________________________________ LSAT Test XVIII Explained: Section II

38 © K A P L A N

• It helps to see that all the statements are of the same type -- each describes anecessary condition: For teachers to be effective, it’s necessary that they help theirstudents become independent learners. For teachers to enable their students to maketheir own decisions, it’s necessary that they have the power to make decisions in theclassroom. It can’t be emphasized enough how important it is to understand thedifference between necessary and sufficient conditions. The testmakers play off thisdistinction all the time in Logical Reasoning stimuli, in Logical Reasoning answerchoices, and in Logic Games setups and rules.

• Notice the words and phrases used in this stimulus to denote necessity: “onlywhen,” “not until X can Y,” and “is essential.”

• Use all of the skills that you acquire in your preparation wherever they apply. Youknow from Logic Games that in a “could be true EXCEPT” question, the choice youseek is the one that’s objectively impossible.

24. (A)Dr. Ruiz wants to make sure the panel examines the issue of second-hand cigarrette smokein an unbiased manner; therefore, he decides Dr. Smith can’t be included in the panel,despite her otherwise good qualifications, because she has expressed strong antismokingviews in public. Connecting this evidence to this conclusion is a fairly clear assumption:Including a person with strong views, like Smith, would damage the ability of the panel toexamine the issue in an unbiased manner. If, as (A) states, a panel composed of qualifiedpeople who have conflicting strong outspoken views is especially likely to arrive at anunbiased conclusion, then Dr. Smith should be a valued member of Dr. Ruiz’ panel.

(B) Au-contraire! (B) supports Dr. Ruiz’ argument against Dr. Smith, by suggesting that Dr.Smith probably won’t fairly evaluate evidence that argues against her position.

(C) So what: Certainly the panel would prefer “lively discussions,” over monotonous dullones, but that’s beside the point; there’s no evidence that lively discussions would be moreor less likely to lead to an unbiased conclusion, which is, after all, what Dr. Ruiz is worriedabout.

(D) So what, the sequel; same concept as above: The fact that people who have expressedstrong views in public are good fund raisers doesn’t show that their presence on a panelhelps the panel arrive at an unbiased conclusion.

(E) Au-contraire! the sequel: The flip side of (B), but with the same result. This supports Dr.Ruiz’ fear that including a person with such strong views as Dr. Smith might inhibit thepanel’s ability to evaluate the issue fairly and in an unbiased manner.

• When an author (or speaker) adopts a course of action, look at the purpose the actionis intended to achieve; a criticism of the course of action should show how it will failto achieve its stated purpose. The purpose of excluding Dr. Smith is to ensure thatthe panel arrives at an unbiased conclusion; any choice that doesn’t address thatissue is irrelevant.

Page 41: 18-PrepTest 18 Explsdl.keywin.org/4/9/49d31fb5129d388a6dd62829ca482add.pdf · 2012. 4. 13. · wrong choices in a “could be false” question are statements that must be true.)

© K A P L A N 39

SECTION III:

READING COMPREHENSION

Page 42: 18-PrepTest 18 Explsdl.keywin.org/4/9/49d31fb5129d388a6dd62829ca482add.pdf · 2012. 4. 13. · wrong choices in a “could be false” question are statements that must be true.)

LSAT PREP _____________________________________________________________ LSAT Test XVIII Explained: Section III

40 © K A P L A N

Passage 1—Law and Literature(Q. 1-7)

Topic and Scope: The law-and-literature movement; specifically, whether this movementhas anything to contribute to the study of law or literature.

Purpose and Main Idea: The author’s purpose is to critique the law-and-literaturemovement. His specific main idea is that this movement simply has nothing to add to thestudy of either law or literature.

Paragraph Structure: ¶1 describes the claims made by the law-and-literature movement:According to its supporters, the movement’s integration of the techniques of literary andlegal analysis has led to conceptual breakthroughs in both literary and legal studies. Laterin the ¶, the author notes that the movement has gained prominence, but the tone of hisremarks already suggests that he’s not a fan.

¶s 2 and 3, as we might have predicted, refute the claims made by movement supportersby citing the views of their most prominent critic, Richard Posner. ¶2 argues that legalanalysis of literary texts that touch on legal matters doesn’t advance the field of literarystudies because these texts don’t really delve into the legal issues that lawyers can shedlight on. ¶3, on the other hand, asserts that the techniques of literary analysis, which focuson broad interpretations of texts, are at odds with the very precise interpretation of lawsthat the field of legal studies is concerned with.

¶4 tries to explain why, in light of his critical views, Posner nevertheless endorses (albeit ina qualified way) the law-and-literature movement. The author speculates that Posner doesso in order to appease movement supporters. Whatever the truth, it becomes crystal clearin this ¶ that the author himself is indeed no fan of the movement.

The Big Picture:

• Notice the classic structure of this passage. ¶1 sets out a couple of claims and theremainder of the passage addresses their validity. Consequently, even though theauthor’s specific main idea isn’t finally revealed until the last paragraph, thispassage is not a bad place to begin work on the section.

• Some of the ideas in this passage—especially in ¶s 2 and 3— are rather abstract.Don’t worry about grasping every nuance of the text; just get the gist of each ¶. Youcan always go back and reread if you need greater insight into a particular idea.

Page 43: 18-PrepTest 18 Explsdl.keywin.org/4/9/49d31fb5129d388a6dd62829ca482add.pdf · 2012. 4. 13. · wrong choices in a “could be false” question are statements that must be true.)

LSAT PREP _____________________________________________________________ LSAT Test XVIII Explained: Section III

© K A P L A N 41

The Questions:

1. (A)This is the only choice that encompasses the passage’s topic, scope, and purpose.

(B) Au contraire. The author contends that mixing legal and literary analytical techniquesbenefits neither legal nor literary studies.

(C) The author examines the law-and-literature movement through the eyes of “its mostdistinguished critic,” Richard Posner. Moreover, only one example of this movement “inpractice” is offered (in ¶2).

(D) Au contraire aussi. The author supports Posner’s critique of the movement. In ¶4, hecalls Posner’s analysis “cogent.”

(E) is beyond the scope of the passage. The author critiques the law-and-literaturemovement’s philosophy, not the “role of literary scholars in providing a broader contextfor legal issues.”

• The correct answer to global questions must be broad enough to cover the contentsof the entire passage, yet also be narrow enough to refer specifically to thosecontents.

2. (E)Although Posner’s analysis of the law-and-literature movement points up its fundamentalweaknesses, he still says that it has “promise.” In other words, he suggests that themovement has some value, despite its shortcomings. Likewise, the salesperson in (E)reasons that a certain product has some value, in spite of its faults.

(A)-(D) In none of these choices does the individual in question display any ambivalencewhatsoever. Each of these scenarios is built around a definite preference—for modernmusic in (A); for the more promising political party in (B); for the better sports team in (C);and for a particular point of view in (D).

• If you don’t immediately see the choice that’s analogous to a situation in the passage,try looking for the choice that’s fundamentally different from the other choices.

3. (A)According to lines 40-41, Posner feels that “legal interpretation aims at discovering a singlemeaning” of the law. Put another way, the legalist approach to interpreting law disavowsmultiple interpretations.

(B) Deconstruction, as lines 41-45 make clear, is a literary technique that runs counter toPosner’s view of the legal approach to interpreting law.

(C) distorts the passage. The legal approach looks for a “single meaning” of the law, so howcould it be responsive to “varying...standards” in its interpretation of the law?

Page 44: 18-PrepTest 18 Explsdl.keywin.org/4/9/49d31fb5129d388a6dd62829ca482add.pdf · 2012. 4. 13. · wrong choices in a “could be false” question are statements that must be true.)

LSAT PREP _____________________________________________________________ LSAT Test XVIII Explained: Section III

42 © K A P L A N

(D) Au contraire. While literary critics, according to Posner, come up with broad andvaried interpretations of texts, the legal approach zeroes in on the specific meaning of texts.

(E) is beyond the scope of the passage. The “social relevance of the legal tradition” is nevermentioned.

• The correct answer to an inference question will never stray very far from the spiritof the text.

4. (C)According to Posner, the reason that legal training doesn’t provide insight into literarytexts is that these texts usually aren’t concerned with “legal questions per se...” Presumably,though, Posner would argue that a legal background might be useful in analyzing aliterary text that revolves around a legal question.

(A), (B), (D), (E) The thrust of ¶2 is that legal training provides no special insight into aliterary text in which the law or the legal system is used solely as a vehicle to symbolize,represent, or illustrate some non-legal issue.

• This question is an excellent example of the importance of grasping the gist ofindividual paragraphs. If you understood the basic idea in ¶2, choice (C) shouldhave leaped off the page.

5. (B)In lines 10-11, the author refers to the law-and-literature movement as a success. In lines 11-13, he defines this success as the movement’s acceptance “in law journals and in leadinglaw schools.”

(A) The phrase “whatever the truth of this last claim...” (lines 9-10) refers directly to theissue of the law-and-literature movement’s impact on the sensitivity of lawyers; and theambivalence of this phrase indicates that the author is by no means convinced that themovement has had a “positive effect on the sensitivity of lawyers.”

(C) and (D) The author’s a harsh critic of the law-and-literature movement: As the passagereveals, he endorses Posner’s view that this movement doesn’t “offer fresh insights intoliterary texts,” (C), and doesn’t “encourage innovative approaches” to studying law andliterature, (D).

(E) distorts the passage. The only point made about the connection between the law-and-literature movement and law journals is that the former has been endorsed by the latter.

• Understanding the context in which a word (or phrase) appears is the key tounlocking “vocabulary-in-context” questions. Reread the lines around the citedword (or phrase) before picking any of the choices.

Page 45: 18-PrepTest 18 Explsdl.keywin.org/4/9/49d31fb5129d388a6dd62829ca482add.pdf · 2012. 4. 13. · wrong choices in a “could be false” question are statements that must be true.)

LSAT PREP _____________________________________________________________ LSAT Test XVIII Explained: Section III

© K A P L A N 43

6. (A)This choice is a nice paraphrase of lines 22-27.

(B) distorts the passage. It’s not that literary texts refrain from using the law to expressimportant ideas; it’s that they don’t pose legal questions.

(C) is beyond the scope of the passage, which never mentions whether lawyers “haveenough literary training to analyze literature competently.”

(D) is an au contraire choice. According to the passage, Posner argues that legal analysis isnot useful to the interpretation of literature because literature focuses primarily on non-legal, as opposed to legal, issues.

(E) The passage never makes any distinction between “contemporary” and non-contemporary literature. Moreover, Posner asserts that legal interpretations are essentiallyirrelevant to all literature.

• The best approach to explicit text questions is to go back to the relevant piece(s) oftext and reread carefully. Don’t answer on a hunch or vague recollection.

7. (A)According to the author, “Posner argues that literary criticism in general aims at exploringrichness and variety of meaning in texts, whereas legal interpretation aims at discovering asingle meaning.” Literary criticism, in other words, can’t be used to interpret law because“the goals of the two disciplines are incompatible.”

(B), (D), and (E) are all beyond the scope of the passage. The text says nothing about thepopularity of the law-and-literature movement within the literary profession, (B); it saysnothing about the interpretation of law “relying heavily” on legal precedent, (D); and itsays nothing about whether legal scholars are willing to adopt the techniques of literarycriticism in their classrooms, (E).

(C) distorts the passage. Posner argues that literary criticism can’t be used to interpret lawbecause literary critics have a different intellectual agenda than legal theorists. Lack ofsophistication in literary methods misses the point.

• Don’t be taken in by choices that use the passage’s language but distort its ideas orare outside its scope.

Page 46: 18-PrepTest 18 Explsdl.keywin.org/4/9/49d31fb5129d388a6dd62829ca482add.pdf · 2012. 4. 13. · wrong choices in a “could be false” question are statements that must be true.)

LSAT PREP _____________________________________________________________ LSAT Test XVIII Explained: Section III

44 © K A P L A N

Passage 2—Scientific Objectivity(Q. 8-15)

Topic and Scope: Scientific objectivity; specifically, whether accepted scientificobservations reflect an objective or subjective view of the world.

Purpose and Main Idea: The author’s purpose is to assess the view of historians of sciencewho assert that accepted scientific observations are based on ideological predispositionsrather than objective reality; his specific main idea is that these historians have adopted anextreme and untenable position.

Paragraph Structure: ¶1 describes the view of these historians, and notes that they haveattempted to strengthen this view by taking a philosophy of science argument to anextreme (that currently accepted scientific observations may ultimately be proven wrongthrough future research). The last sentence of this ¶ is particularly important because theauthor’s voice enters the picture: He labels these historians as “extremist.” Given thisattitude, it’s predictable that the rest of the passage is going to be a critique of their view.

¶2 bears out this prediction. The author demonstrates that their view is “deeplyimplausible” by citing a couple of scientific observations that have proven to be accuratereflections of the real world. In ¶3, the author qualifies his position somewhat by notingthat, while scientific observations can certainly be accurate reflections of the real world,such is not always the case; often, these scientific observations occur only after a longprocess of discovery in which previously accepted observations are shown to be false.

Finally, in ¶4, the author summarizes his view: Many scientific observations reflectobjective reality, contrary to what the “recent generation of historians of science” say.

The Big Picture:

• A second rather abstract passage. But, like the first passage, it’s not too difficult. Theauthor’s take on the objective vs. subjective debate is made clear by the end of thefirst paragraph, and the rest of the passage simply fleshes out his view in moredetail.

• Most LSAT science passages deal with “concrete” subject matter—a theory, anexperiment, a scenario, etc. Occasionally, however, they are more philosophical innature, like the present one. On test day, don’t be thrown if you run into this type ofscience passage. Fundamentally, it’s no more difficult than the more traditional typeof science passage.

Page 47: 18-PrepTest 18 Explsdl.keywin.org/4/9/49d31fb5129d388a6dd62829ca482add.pdf · 2012. 4. 13. · wrong choices in a “could be false” question are statements that must be true.)

LSAT PREP _____________________________________________________________ LSAT Test XVIII Explained: Section III

© K A P L A N 45

The Questions:

8. (C)This choice makes essentially the same point as lines 32-39, in which the author asserts thatscientific truths eventually emerge out of a complex process of discovery.

(A) Au contraire. According to the author, what is “implausible” is the position ofhistorians of science who think that scientific truths rest on ideological bias rather observedreality.

(B) is too extreme. While the author feels that many scientific truths are falsifiable (lines 40-43), he does cite some truths that he believes will never be falsified (lines 24-31).

(D) Another au contraire choice. Lines 32-39 indicate that the author thinks that scientifictruths often do emerge from “creative processes.”

(E) is the view of the historians of science whom the author attacks.

• Choices that contradict or distort the thrust of the passage are especially common ininference questions.

9. (A)In lines 43-44, Newton’s updating of Kepler’s ideas is cited as an example of a generalscientific phenomenon: newer scientific ideas correcting older ones.

(B) refers to an unrelated detail in lines 37-39.

(C) Just the reverse: Kepler’s ideas were originally thought to be accurate, but later werecorrected by Newton.

(D) refers to the view of historians of science. But the Kepler example comes up in thecontext of a discussion of the author’s quite different view.

(E) Although Newton did correct Kepler’s ideas, there is nothing in the passage to indicatethat he intentionally set out to do so.

• Wrong choices are often wrong for fairly subtle reasons—like choice (E) here. So,read each choice fully and carefully before settling on any one of them.

Page 48: 18-PrepTest 18 Explsdl.keywin.org/4/9/49d31fb5129d388a6dd62829ca482add.pdf · 2012. 4. 13. · wrong choices in a “could be false” question are statements that must be true.)

LSAT PREP _____________________________________________________________ LSAT Test XVIII Explained: Section III

46 © K A P L A N

10. (C)In ¶2, the author strongly implies that scientific observations are accurate reflections of thereal world, and even cites a couple of examples to reinforce the point. In ¶3, however, hehedges his position; uncovering genuine scientific truths, he now suggests, is actually amore complicated business than he previously implied.

(A) What conflicting explanations?

(B) focuses on the wrong ¶. A new field of study is mentioned in ¶4.

(D) What theoretical question?

(E) also focuses on the wrong ¶. The author attacks the view of the recent crop of historiansof science in ¶2.

• When a question asks about the primary purpose or organization of a specific ¶,watch out for choices that reflect the main idea or organization of some other ¶.

11. (E)These words are used in the context of an attack on those historians of science who arguethat scientific observations aren’t based on objective reality. Hence, the words serve todiscredit their view.

(A) “Earlier notions about the composition of water” are never mentioned in the passage.The author refers to current beliefs about water’s composition simply in order to make thepoint that some scientific truths will never be falsified, not to contrast it with outdatednotions.

(B) The author is not critical of philosophers of science.

(C) and (D) are au contraire choices. The author supports the traditional view of scientificobjectivity, (C), and is in sympathy with those who hold this view, (D).

• Whenever a question stem cites a line reference, read the lines around that referencecarefully. That’s where the answer lies.

Page 49: 18-PrepTest 18 Explsdl.keywin.org/4/9/49d31fb5129d388a6dd62829ca482add.pdf · 2012. 4. 13. · wrong choices in a “could be false” question are statements that must be true.)

LSAT PREP _____________________________________________________________ LSAT Test XVIII Explained: Section III

© K A P L A N 47

12. (B)In line 12, the author describes “certain philosophers of science” and recent historians ofscience as “allies.” In line 20, he argues that it’s an “easy step” from the view of thesephilosophers to the view of these historians. Thus, the author would contend that theirviews have elements in common.

(A) is too extreme. Saying that it’s an “easy step” from one view to the other is not the sameas saying that they’re “difficult to differentiate.”

(C) Just because some philosophers and historians have similar views in certain respectsdoesn’t mean that the historians’ view stems from that of the philosophers. Indeed, theauthor argues that the philosophers would not agree with the historians’ general line ofthinking (lines 17-19).

(D) Only the historians’ view emphasizes “the rhetorical power of scientists.”

(E) The passage doesn’t say or suggest that the historians would acknowledge any suchthing.

• Always be on the lookout for choices that go beyond what the text allows you toinfer—like choices (A), (C), and (E) here.

13. (D)The author labels the historians’ view as “extremist” and “implausible,” but in ¶3,particularly the last sentence, he hints that they do have some valid ideas.

(A) is too extreme. The author does give them a little credit, after all.

(B) distorts the passage. It’s the process of scientific discovery that the author thinks can be“rewardingly studied” as a social phenomenon, not the historians’ view.

(C) Au contraire: The author thinks that the historians are on to something in their analysisof “scientific groups.”

(E) This is a “half-right, half-wrong” choice. The author concedes that the historians’ viewis popular today (lines 22-24), but he doesn’t say that it’ll soon be overturned.

• “Pre-phrasing” the author’s opinion of the historians’ view might have helped you toeliminate choices (A) and (E), which are plausible-sounding choices that aren’t quiteright.

Page 50: 18-PrepTest 18 Explsdl.keywin.org/4/9/49d31fb5129d388a6dd62829ca482add.pdf · 2012. 4. 13. · wrong choices in a “could be false” question are statements that must be true.)

LSAT PREP _____________________________________________________________ LSAT Test XVIII Explained: Section III

48 © K A P L A N

14. (A)In the last sentence of the passage, the author recommends that historians of scienceundertake a new research project. In other words, he “offers a prescription.”

(B) What paradox?

(C) What prediction?

(D) In ¶4, the author reaffirms his commitment to his view; he doesn’t concede anything tothe historians of science.

(E) What objections?

• Don’t linger over the choices. Remember, only one is categorically correct. Onceyou’ve found it, quickly eliminate the others.

15. (D)The “thesis” in question refers to the view of historians of science. The author has anegative attitude toward this thesis, calling it “extremist” (line 20) and “implausible” (line24).

(A) “Biases” (line 5) and “rhetorical” (line 6) are words that are used to describe thehistorians’ thesis, not the author’s attitude toward that thesis.

(B) “Wield” (line 7) is another word that is used to describe the historians’ thesis, while“falsification” (line 17) is a word used to describe the ideas of philosophers of sciencewhose view lines up with the historians’ thesis.

(C) “Conjectures” (line 16) and “truck with” (line 19) don’t convey any sense of negativity.Besides, neither of these words is used in connection with the author’s attack on thehistorians’ thesis.

(E) Although “naive” (line 35) and “errors” (line 42) certainly have a negative ring to them,the author doesn’t use these words in his criticism of the historians’ thesis. Rather, they areused in the context of an explanation of his own view about how scientific truths areestablished.

• If you picked up on the author’s negative attitude toward the historians’ thesis, thenegative words in choice (D) should have set off an immediate alarm in your head.

Page 51: 18-PrepTest 18 Explsdl.keywin.org/4/9/49d31fb5129d388a6dd62829ca482add.pdf · 2012. 4. 13. · wrong choices in a “could be false” question are statements that must be true.)

LSAT PREP _____________________________________________________________ LSAT Test XVIII Explained: Section III

© K A P L A N 49

Passage 3—Native Americans(Q. 16-21)

Topic and Scope: Cherokee culture; specifically, the extent to which the Cherokee tribeabsorbed Euro-American culture during the 1820s.

Purpose and Main Idea: The author’s purpose is to present two views of Cherokeecultural assimilation and to determine which is more convincing; the author’s specificmain point is that McLoughlin’s view is more compelling than the traditional view, eventhough it is flawed.

Paragraph Structure: ¶1 paints the traditional view of Cherokee cultural assimilation inthe 1820s, relating that a combination of missionaries and part-Cherokee intermediaries, aswell as the US government, “imposed” Euro-American culture on the Cherokees.

¶2, in contrast, presents McLoughlin’s “revisionist” view that traditional Cherokee cultureflourished alongside, and in reaction to, Euro-American cultural reforms. ¶3 amplifies hisview, noting that traditionalist Cherokee accepted Euro-American cultural reforms tostrengthen their place in American society, but incorporated these reforms into their owndistinctive culture.

In ¶4, the author basically endorses McLoughlin’s view of Cherokee cultural identity overthe traditional view, though he also points out that McLoughlin overlooked the impact ofpre-nineteenth century Euro-American culture on Cherokee culture.

The Big Picture:

• Another passage with a classic structure—in this case, a classic “book review”passage structure. ¶1 sets out the traditional view, ¶s 2 and 3 lay out the “revisionist”view, and ¶4 presents the author’s opinion about which is more compelling. Thisneat, predictable structure makes this passage an excellent place to begin work onthe section, even though the author’s view isn’t fully apparent until the final ¶.

• Book review passages generally contain a lot of factual detail. Don’t worry aboutassimilating all of it during a first read through. Details can always be looked upwhen necessary.

Page 52: 18-PrepTest 18 Explsdl.keywin.org/4/9/49d31fb5129d388a6dd62829ca482add.pdf · 2012. 4. 13. · wrong choices in a “could be false” question are statements that must be true.)

LSAT PREP _____________________________________________________________ LSAT Test XVIII Explained: Section III

50 © K A P L A N

The Questions:

16. (B)This choice is the only one that captures the author’s topic, scope, and purpose. Note howit gets at the traditional vs. revisionist debate that is at the heart of the passage.

(A) The second part of this choice focuses on a detail in ¶4, while the first part is far toonegative in tone: Though the author concedes that McLoughlin’s work is flawed, he’sessentially in sympathy with that work.

(C) Again, the second part of the choice plays on a detail, in ¶2. Furthermore, the first partof the choice distorts McLoughlin’s view, which acknowledges some missionary influenceon Cherokee culture.

(D) is a “half-right, half-wrong” choice. The first half of the choice says exactly what isstated in lines 15-17, but the second half is too extreme. According to McLoughlin, at leastsome of the changes in Cherokee culture were the product of Euro-American culturalimports.

(E) First, the author doesn’t accuse McLoughlin of having overlooked “relevant factors inCherokee cultural change.” Second, McLoughlin argues that, while Cherokee culturalchange was spurred to some extent by outside influences, much of the change was drivenby the Cherokee themselves.

• In global questions, don’t be taken in by choices that focus on details or thatexaggerate ideas in the passage.

17. (D)Lines 44-49 show that the Cherokee council was able to regulate the activities of Whitepeople living on Cherokee land, while lines 10-14 reveal that the Cherokee did not entirelygovern their own affairs.

(A) is beyond the scope of the passage, which doesn’t mention anything about howmembers of the council were chosen.

(B) The phrase “most [as opposed to all] members of the Cherokee council” (line 45) seemsto suggest that unanimity wasn’t necessary for a policy to go into effect.

(C) Au contraire. Lines 10-14 suggest just the opposite.

(E) Lines 45-46 make it clear that the council consisted of both traditionalist andacculturating Cherokee, but the text doesn’t say whether the mix on the council reflectedthe mix among the general Cherokee populace.

• Never endorse a choice just because its lingo is similar to the passage’s lingo.

Page 53: 18-PrepTest 18 Explsdl.keywin.org/4/9/49d31fb5129d388a6dd62829ca482add.pdf · 2012. 4. 13. · wrong choices in a “could be false” question are statements that must be true.)

LSAT PREP _____________________________________________________________ LSAT Test XVIII Explained: Section III

© K A P L A N 51

18. (E)Lines 35-44 indicate that traditionalist Cherokee accepted the reforms for utilitarianpurposes: They believed that these reforms would make the tribe less ripe for exploitationby the majority White society.

(A), (C), (D) Nothing in the passage suggests either that the “acculturating elite” becamemore vocal in the 1820s or that traditionalist Cherokee agreed to the reforms to placate thiselite, (A). Indeed, the passage speaks of “intratribal tensions” between traditionalist andacculturating Cherokee over the reforms. Similarly, nothing in the passage suggests thattraditionalist Cherokee believed that the reforms were “inevitable,” (C), or an extension ofexisting Cherokee culture, (D).

(B) Traditionalist Cherokee accepted the reforms because they thought that by doing sothey would be helping to preserve traditional Cherokee culture.

• If you were attentive to the thrust of individual ¶s, choice (C) should haveimmediately caught your eye.

19. (C)Lines 21-26 say that the missionary “tendency to cater to the interests of anacculturating...elite...at the expense of the more traditionalist...majority created greatintratribal tensions.” The result of these tensions, according to lines 26-31, was thattraditionalist Cherokee revived traditional religious beliefs to counter the elite’s reforms.

(A) For starters, the passage never says that the missionaries gained “converts,” just that anacculturating elite tended to approve of their influence on Cherokee society. Moreover,there’s nothing in the passage to suggest that the rate at which this elite grew increased asthe 1820s progressed.

(B) If the traditionalist majority felt that the reforms would corrupt Cherokee culture, thenwhy did they ultimately accept them? The traditionalists thought that they could controlthe negative effects of missionary-inspired reforms, while using these reforms to the tribe’sadvantage.

(D) The use of the word “many” (line 47) suggests that traditionalists felt that more thanjust “some” of the reforms would be beneficial to the tribe in general.

(E) Although the passage says that the “elite initiated reforms...to legitimize their own”status, it doesn’t suggest that this behavior manifested itself in an attempt to take politicalcontrol of the Cherokee council.

• The answer to explicit text questions is generally to be found in the lines around theinformation cited in the question stem itself. It’s rarely necessary to scan the entirepassage for the answer.

Page 54: 18-PrepTest 18 Explsdl.keywin.org/4/9/49d31fb5129d388a6dd62829ca482add.pdf · 2012. 4. 13. · wrong choices in a “could be false” question are statements that must be true.)

LSAT PREP _____________________________________________________________ LSAT Test XVIII Explained: Section III

52 © K A P L A N

20. (D)In ¶2, McLoughlin links the revival of traditional Cherokee religious beliefs and practicesto the activities of missionaries. This cause-effect relationship, of course, would beundermined if it were shown to be true that the religious revival had actually begun beforethe missionaries appeared on the scene.

(A) Since the traditionalist and acculturating Cherokee on the council were in basicagreement about both the reforms and missionary and settler behavior the balance betweenthem on the council had no bearing on these issues. Thus, McLoughlin’s account wouldnot be undermined if it were true that the majority of the council members were in facttraditionalists.

(B) McLoughlin acknowledges that outside forces, like the government, tried to shapeCherokee society to their liking.

(C) McLoughlin himself points out that the missionaries encouraged the elite to press forthese reforms.

(E) McLoughlin notes that the acculturating elite had self-serving reasons for pressing forthe reforms.

• In Strengthen/Weaken questions, read the stem very carefully in order to determinewhether it’s asking you to strengthen or weaken an argument, and be careful that thechoice you endorse does not do the opposite.

21. (B)In the first sentence of ¶4, the author criticizes McLoughlin for overlooking pre-19th-century Euro-American “influences” on Cherokee culture. In other words, the authorthinks that a “process of acculturalization” was already underway among the Cherokeebefore the 1820s.

(A) is beyond the scope of the passage, which doesn’t talk about acculturalization after the1820s.

(C) According to the author, the Cherokee had contact with “White resident traders andneighbors” before the 1820s. There’s no mention of contact with missionaries before thisdecade.

(D) Although the author acknowledges that the US government encouragedacculturalization, he doesn’t say or suggest that the government was crucial to the process.

(E) distorts a detail in ¶4, which says that “White traders” may have had an influence on theCherokee tribe before the 1820s.

• You could have saved yourself a lot of time searching for the answer to this questionif you remembered that the author’s take on McLoughlin’s account appears in ¶4.

Page 55: 18-PrepTest 18 Explsdl.keywin.org/4/9/49d31fb5129d388a6dd62829ca482add.pdf · 2012. 4. 13. · wrong choices in a “could be false” question are statements that must be true.)

LSAT PREP _____________________________________________________________ LSAT Test XVIII Explained: Section III

© K A P L A N 53

Passage 4—Luminists(Q. 22-28)

Topic and Scope: Luminist painting; specifically, the interpretation of Luminist painting.

Purpose and Main Idea: The author’s purpose is to offer a non-traditional interpretation ofLuminist painting by examining the work of Fitz Hugh Lane; the author’s specific mainidea is that, contrary to the accepted view, Luminist painting doesn’t present a spiritualand mystical view of untamed nature.

Paragraph Structure: ¶1 presents the accepted view of Luminist painting. In ¶2, the authorprovides a counterinterpretation, arguing that Luminist works actually portray man’sconquest and exploitation of nature.

¶s 3 and 4 illustrate the author’s view through a detailed examination of Lane’s landscapes.Essentially, what the author says is that Lane’s harbor views are meant to portray man’sdomestication and exploitation of the sea for the purposes of expanding commerce.

The Big Picture:

• Yet another passage with a classic structure. ¶1 provides the accepted view of thetopic, while ¶s 2, 3, and 4 present a counterinterpretation through an extendedexample.

• Since topic, scope, and purpose are evident early on, this passage is certainly onethat should be tackled earlier rather than later in the section.

The Questions:

22. (B)¶s 2, 3, and 4 describe the author’s perspective about Luminist painting, principally byanalyzing the work of one artist, Fitz Hugh Lane.

(A) The allegedly spiritual and mystical (or religious) nature of Luminist painting isdiscussed only in ¶1. Besides, this passage isn’t about art in general.

(C) plays on a detail, mainly in ¶4. Luminist painting, not the sea, is the central theme ofthis passage.

(D) focuses on a detail in ¶1.

(E) is beyond the scope of the passage, which is about one particular school of 19th-centuryAmerican landscape painters. The text isn’t about 19th-century landscape painters ingeneral.

Page 56: 18-PrepTest 18 Explsdl.keywin.org/4/9/49d31fb5129d388a6dd62829ca482add.pdf · 2012. 4. 13. · wrong choices in a “could be false” question are statements that must be true.)

LSAT PREP _____________________________________________________________ LSAT Test XVIII Explained: Section III

54 © K A P L A N

• Be on the lookout for choices that violate the scope of the text when dealing withglobal questions. Remember, the correct choice must make some sort of reference tothe specific issue at hand.

23. (E)In lines 43-45, the author says, “For Lane the sea is...a canal or a trade route for commercialactivity, not a free, powerful element.” In line 46, he says “For Lane, nature is subdued....”In line 49, he says, “I consider Lane’s sea simply an environment for human activity—nature no longer inviolate.” Moreover, elsewhere in the passage, the author makes it clearthat he believes that Luminist paintings portray man’s dominance of nature.

(A), (B), (D) “Wild and unexplored,” (A), “idealized and distant,” (B), and “difficult tounderstand,” (D), are phrases that evoke sentiments that are the opposite of those that theauthor holds toward Lane’s work.

(C) “Continually changing” is a phrase that the author might apply to pre-Luministportrayals of nature.

• In any passage that contains multiple perspectives, it’s important to be clear aboutwho advocates each of them—the questions will certainly test to see that you’vepicked up this information.

24. (D)In the last sentence of ¶3, the author mentions that Lane lived near and travelled to variouscommercial ports. In ¶4, the author suggests that this proximity accounts for Lane’spropensity to paint harbor scenes that reflect business themes.

(A) This view about the connection between religion and art is held by the unnamed artcritic mentioned in lines 23-27, not by the author.

(B) is beyond the scope of the passage, which doesn’t concern itself with “viewer interest.”Besides, Luminist paintings are idealized, not highly detailed, landscapes.

(C) The passage doesn’t say that commerce is an unusual artistic theme. Nor does it saythat artists interested in this theme must travel and observe it widely. Lane did, but thatdoesn’t mean that others must do (or have done) likewise.

(E) is also beyond the scope of the passage, which isn’t about “popular works of art” andtheir themes. It’s about the interpretation of one particular school of painting.

• Quickly skim and dismiss choices that fall outside of the passage’s scope—like (B),(C), and (E) here.

Page 57: 18-PrepTest 18 Explsdl.keywin.org/4/9/49d31fb5129d388a6dd62829ca482add.pdf · 2012. 4. 13. · wrong choices in a “could be false” question are statements that must be true.)

LSAT PREP _____________________________________________________________ LSAT Test XVIII Explained: Section III

© K A P L A N 55

25. (C)According to the author, the accepted view of Luminist works is incorrect because itdoesn’t interpret the portrayal of nature in these works as conquered and exploited byman.

(A) Au contraire. The accepted view of Luminist painting concentrates on its use of light.

(B) Those who adhere to the accepted view see Luminist paintings as “tranquil.”

(D) and (E) It’s the author who focuses on the practical side, (D), and subject matter, (E), ofLuminist works.

• In explicit text questions, be on the lookout for au contraire choices.

26. (B)Most of ¶s 3 and 4 explore the connection between the sea and commerce in Lane’spaintings.

(A), (C) “Exploration,” (A), and “canals,” (C), are words that the author uses as part of hisdiscussion of the link between the sea and commerce in Lane’s work. The author doesn’tcontend that Lane himself associated the sea with either of these things.

(D), (E) “Idealism,” (D), and “mysticism,” (E), are terms that those who subscribe to thetraditional view of Luminist painting apply to this school in general.

• Never pick a choice simply because it uses the same words or phrases that thepassage uses.

27. (B)¶1 lays out the accepted interpretation of Luminist painting. ¶s 2, 3, and 4, in contrast, aredevoted to presenting an alternate interpretation, which, according to the author, is moreaccurate than the accepted interpretation.

(A) First, the passage doesn’t discuss any “theory.” Second, the accepted interpretation ofLuminist painting isn’t “new.”

(C), (D) The passage is primarily about the author’s interpretation of Luminist painting,not the attitudes, (C), or interpretations, (D), of critics in general. Both of these choices playon passage details.

(E) The author briefly sets out what is considered to be a wrongheaded interpretation ofLuminist painting, but the history of that interpretation is never probed.

• In global questions, abstractly-phrased choices needn’t pose a problem. Simplysummarize the contents of each ¶ and look for the corresponding choice.

Page 58: 18-PrepTest 18 Explsdl.keywin.org/4/9/49d31fb5129d388a6dd62829ca482add.pdf · 2012. 4. 13. · wrong choices in a “could be false” question are statements that must be true.)

LSAT PREP _____________________________________________________________ LSAT Test XVIII Explained: Section III

56 © K A P L A N

28. (E)The Contrast Keyword “however” (line 28), which comes immediately after the quote inquestion, indicates that the author has included this quote in order to “illustratea...misconception concerning a characteristic of Lane’s paintings.” In particular, the authorclaims that Lane’s paintings reflect a growth in commerce rather than widespread religiousfervor.

(A) is beyond the scope of the passage, which doesn’t refer to non-Luminist painting.

(B) Au contraire. The author includes the quote to help make the point that Lane wasinterested in commerce, not spiritualism.

(C) The quote is included to point out a “cultural factor” that the author feels didn’tinfluence Luminist painting.

(D) The author doesn’t discuss this correlation. Indeed, he denies an explicit link betweenLuminist painting and spiritual fervor.

• Questions often ask about the purpose, not the substance, of a detail—that’s onemajor reason why you’ve got to read for “¶ sense.”

Page 59: 18-PrepTest 18 Explsdl.keywin.org/4/9/49d31fb5129d388a6dd62829ca482add.pdf · 2012. 4. 13. · wrong choices in a “could be false” question are statements that must be true.)

© K A P L A N 57

SECTION IV:

LOGICAL REASONING

Page 60: 18-PrepTest 18 Explsdl.keywin.org/4/9/49d31fb5129d388a6dd62829ca482add.pdf · 2012. 4. 13. · wrong choices in a “could be false” question are statements that must be true.)

LSAT PREP _____________________________________________________________ LSAT Test XVIII Explained: Section IV

58 © K A P L A N

NOTE: In some versions of this PrepTest, there are two typos in the Law Services answerkey. On this Logical Reasoning Section 4, the answer to Question 8 should read (B) and theanswer to Question 9 should read (C).

1. (B)What’s “foolish” about relying on assurances from biotech companies that their productsare safe? (B) explains: Those who are planning to benefit financially shouldn’t be the onlyadjudicator of product safety. This makes sense in light of the stimulus, and justifies theconclusion’s recommendation for an additional—independent—adjudicator.

(A), depending on how you look at it, either contradicts the stimulus (“voluntaryguidelines” are emphatically INsufficient, according to the author) or outside the scope(it’s regulation of the potentially dangerous products, not the safe ones, that is at issue here).

(C) Nothing in the stimulus mentions, or even relates to, allowing harmful methods toavoid even more harmful effects.

(D) is off the point—the issue is not what a company is obligated to do, but whatgovernmental regulation should require over and above what a company is willing to do.The biotech companies are willing to assure us that their gene splicing is safe. But is it?

(E) The first seven words of (E) look promising, but it’s all downhill from there: Therecommendation is for an oversight board composed of scientists, the very group that (E)points to as being inadequate to the task.

• “Principle” questions hinge largely on topic and scope, and can be managed if youcarefully match up each element of the choice with the essentials of the givensituation. Note carefully here how (A), (D), and (E), in particular, deviate from thestimulus’ scope.

2. (B)A heads-up attitude and structural keywords are all you need to recognize that sentence 2,or choice (B), is the conclusion. Sentence 1 elicits no reaction from the reader except: Somoney’s tight and spending needs to be cut; fair enough. And “nevertheless” does implythat the previous sentence is indeed a given. But sentence 2’s claim that cutting the Zoobudget “is false economy” requires you to respond: How so? Where’s your evidence? Nextwe get sentence 3; then “Furthermore” and sentence 4; and then “Finally” and sentence 5.Even if you read no more than that, you have to recognize that structurally speaking,sentences 3-5 are the given reasons—evidence—for sentence 2. Nothing else is possible. Andindeed, sentences 3-5 do give different reasons as to why cutting the Zoo budget shouldnot be part of that overall spending-reduction program called for at the outset.

(A) is neither stated nor implied, even by sentence 1. There may well be other measuresneeded.

(C), (D), and (E) each cite an aspect of one piece of evidence: sentences 3, 4, and 5,respectively.

Page 61: 18-PrepTest 18 Explsdl.keywin.org/4/9/49d31fb5129d388a6dd62829ca482add.pdf · 2012. 4. 13. · wrong choices in a “could be false” question are statements that must be true.)

LSAT PREP _____________________________________________________________ LSAT Test XVIII Explained: Section IV

© K A P L A N 59

• Even when a question doesn’t explicitly demand the argument’s conclusion, youalways need to locate it. And in the absence of obvious Keywords like “therefore”and “clearly,” you can locate it by looking for an idea that by its very nature demandssupporting evidence. If the other sentences in the stimulus provide that support, asthey do here, then you can be sure that you’ve found the conclusion.

• “Finally,” in the sense of “in the end,” is rarely used as a Conclusion Keyword. Mostauthors use “finally” to indicate the last member of a series, so it’s what we’ve calleda Sequence Keyword, one that tells you that there’s some kind of order at work.

3. (E)This one becomes a slam dunk if you simply notice that the stimulus conclusion is aprediction of what will occur, and only (E) offers that. Otherwise, you need to follow theauthor’s misunderstanding of sufficiency and necessity. Being handled as a kitten isnecessary for winning a cat’s affection, says the first sentence, but we know that it need notbe sufficient for same. Yet the author thinks it is: She thinks that the kitten’s having beenhandled is enough to confirm that the kitten will like people. In the same way, beingpressed firmly into the ground is but necessary for good germination, but the conclusiondecides that it’s sufficient for same.

The four wrong choices all start in a way parallel to the stimulus’ first sentence, but all fourgo on to demonstrate proper, not flawed, logic:

(A),(D) Each reasons, correctly, that since a condition (two months’ chilling/growing in abog) that’s necessary for a result (flowers produced/cranberries thriving) doesn’t exist, theresult can’t exist.

(B) and (C) are logically identical in the form of “No X unless Y; X, therefore Y. If theevidence and conclusion in either choice’s second sentence were reversed, we’d haveanother correct answer.

• An “unless” clause is generally used to cite an exception to a state of affairs, as forinstance: “You can’t swim in this pool unless you wear a swimsuit.” The “unless”clause thus describes a condition (the swimsuit) that’s necessary for a result(swimming) but it’s not sufficient for that result: One could be barred from the poolon other grounds, such as horsing around or non-payment of a pool fee. So all wecan infer is that if one is swimming, then one is wearing a swimsuit (thatcorresponds to choices (B) and (C), above); and contrapositively, if one lacks aswimsuit, then one cannot swim (that’s (A) and (D), above.) Correct choice (E) takesthe form of “If you wear a swimsuit, you can swim.” Not true—but it is parallel to thestimulus.

4. (C)Two “however” Keywords in one paragraph? Awkward but manageable. The topic iswhere and when higher primates emerged. Was it 30 million years ago in Egypt, as hadlong been thought? Or 10 million years earlier than that in Burma? A recently foundjawbone fragment supports the latter theory, but we’re told that that theory is “premature.”

Page 62: 18-PrepTest 18 Explsdl.keywin.org/4/9/49d31fb5129d388a6dd62829ca482add.pdf · 2012. 4. 13. · wrong choices in a “could be false” question are statements that must be true.)

LSAT PREP _____________________________________________________________ LSAT Test XVIII Explained: Section IV

60 © K A P L A N

This fill-in is a tough one to pre-phrase, but once you saw (C) we hope you pounced on it.The Burma theory is only based on the jawbone fragment and if, as (C) says, such a fragmentcan’t be conclusive, then one clearly cannot rush to affirm that theory. Doing so in theabsence of additional, more conclusive evidence would be. . . well, “premature.”

(A) is chock full of scope shifts: The number of primate species existing in either country isirrelevant to the question of the species’ origin, and the stimulus is concerned with higherprimates only, not (A)’s primates in general.

(B) The authenticity of the age estimates isn’t at issue. If anything, independentconfirmation of the fragment’s age tends to strengthen it as evidence; it hardly sends up awarning flag about “premature” reasoning.

(D) A claim about what experts do or do not believe has nothing necessarily to do withwhat is or is not true. Who cares if experts doubt both theories, as (D) says? One of thetheories could still hold water, depending on the given evidence. A perennial LSAT wronganswer.

(E) This may supercede the evidence for the claim, but it doesn’t show the claim itself (thatthe “earliest higher primates originated in Burma”) to be premature. In other words, thisinformation, if true, supplies additional evidence that the claim is in fact on the mark.

• Pre-phrasing is a great time saver when it pays off, but a few questions simply aren’tvulnerable to it. Don’t strain and invest a lot of time in pre-phrasing an answerchoice in vain. Use your instincts. If nothing comes to you in the first few seconds,feel free to jump into the choices and evaluate them—in whatever order you see fit.

• The ability to notice scope shifts is paramount in Logical Reasoning. Choice (A)contains two classic examples—primates vs. higher primates; number of species vs.origin of species (not to be confused with Darwin)—and if you missed them, youneed to pay more attention to details so that you’re able to see when a subtle shift inscope has occurred.

• Wrong answers like (D) might be called “improper appeal to authority.” (See theexplanation for Section II, Q. 5, for a definition.) Beware of choices suggesting thatan “expert’s” opinion on an idea has any impact on the truth or falsity of that idea.

5. (C)Some deciphering is needed here, starting with the unusual stem. “Substituting a reason”must mean replacing the given evidence with new evidence, and “still preserving theforce” means that the argument has to remain as strong as it was before the switch. (Inessence, the testmakers are asking for parallel evidence.) The three sentences act respectivelyas a factual claim, the conclusion, and the evidence, with the “After all” Keyword phrase aloud signal of the author’s support for his position.

Page 63: 18-PrepTest 18 Explsdl.keywin.org/4/9/49d31fb5129d388a6dd62829ca482add.pdf · 2012. 4. 13. · wrong choices in a “could be false” question are statements that must be true.)

LSAT PREP _____________________________________________________________ LSAT Test XVIII Explained: Section IV

© K A P L A N 61

That position—the conclusion—is that despite various anticipations of chaos and utopia, acentury’s end is usually a letdown. Why? The gist of the evidence is that the concept of“century” is essentially meaningless, since it’s not intrinsic to human life but is justassociated with one of many calendars around. And in the same way, (C) hints at themeaninglessness of the “100 years” concept by pointing out that it’s just based on one ofmany number systems around. Either way, the argument remains the same: Since theconcept of “century” is superficial or cosmetic, it’s small wonder that the expectations ofgreat change fall flat.

(A) speaks to the impossibility of anticipating both chaos and utopia at once, whichsidesteps the real question of why neither anticipation tends to come true at century’s end.Insert this sentence after “After all,” and you’ll see how radically it departs from the scope.

(B), like (A), has a skewed focus on the side issue of the contrast between the two excitedways in which people anticipate a century’s end. Also, (B) tends to affirm those excitedreactions instead of supporting the thesis that “both reactions [tend] to be misplaced.”

(D) hints at a paradox that’s interesting but at best tangential: (D)’s evidence is aboutpeople’s behavior and its paradoxical effect on attitudes, while we need evidence about theirrelevancy of the concept of “century.” Moreover, the reduced likelihood of “somethingextraordinary” happening doesn’t mean that the extraordinary does not nevertheless occur.But the stimulus needs evidence clearly suggesting why nothing out of the ordinaryhappens.

(E)’s focus on learning from mistakes is simply baffling in this context.

• Even after creating thousands of LSAT questions, the test writers can still find a wayto throw you the occasional curve ball. When a stem seems unfamiliar, it’s crucialthat you take some pains to take it apart and translate it. See the bullet point forQuestion 6, below, for another hint.

6. (D)An oddly-worded question stem whose gist is: What can be inferred from the paragraph?The terse statement that people can be “unduly influenced” by subliminal, backwardspoken messages on certain records requires that people can in fact understand suchmessages. . . assuming that conscious understanding (as opposed to unconsciousinfluence) is also necessary, and (D)’s “if” clause covers that.

(A)’s requirement that the spoken messages drown out the music need not be so; the claimcould remain sound even if the spoken words were as loud as, or somewhat softer than, themusic.

(B) is wrong because the claim does not hinge on the retention of the music’s musicalqualities, whatever that means. The qualities of the music aren’t part of the scope.

(C) seems to allude to the reasons why people might be “unduly influenced” in the way thestimulus describes, but those reasons aren’t necessarily so. Maybe this weird Svengali-likeinfluence can work with any music.

Page 64: 18-PrepTest 18 Explsdl.keywin.org/4/9/49d31fb5129d388a6dd62829ca482add.pdf · 2012. 4. 13. · wrong choices in a “could be false” question are statements that must be true.)

LSAT PREP _____________________________________________________________ LSAT Test XVIII Explained: Section IV

62 © K A P L A N

(E) If the influence the stimulus describes can take place even when listeners are onlypaying slight attention—and there’s no reason to believe that it can’t—then (E) certainly isnot a logical consequence of the claim.

• When you encounter a question stem with unusual wording, think about the morecommon question types and relate them to the one you’re now reading; maybe you’llhit on what the question is really asking. Here, for instance, even if you didn’t seethat a logical “consequence of [a] view” means something that must be true—hence,an inference—you might have come to that realization by running through a mentallist of the common LR types.

7. (C)The advertisement places great weight on the fact that 80 percent of the people who test-drive a Zenith wind up buying one; the implication is that this is an extraordinarily highpercentage, and that this shows the Zenith is an extraordinarily fine car. (C) throws doubton that implication by giving an alternate reason for the high percentage of test-driverswho buy the Zenith; in “overwhelming proportions,” people who test-drive a car havealready decided to buy it (barring the discovery of a major problem). If (C) is true, thennothing in the advertisement really implies that the Zenith is an unusually impressive car.

(A) suggests that a “drive around the block” literally might not be enough to convince aperson, but it in no way affects the ad’s implication that the Zenith must be an unusuallyfine car because 80 percent of test-drivers go on to buy it.

(B) What does this have to do with quality? (B) does nothing to cast doubt on theconclusion that the satisfaction of test-drivers shows that the Zenith is a good car.

(D) Logically speaking, it doesn’t matter at all if people wait a day, a week, or a monthbefore signing on the dotted line, as long as they actually buy the car.

(E) ignores the issue of test-driver satisfaction altogether; the fact that a minor part breaksin “some” Zenith cars doesn’t cast much doubt on the car’s quality, especially since dealersare ready to replace the part at no cost. (Sounds like a high-class operation.)

• Pay close attention to the more complex stems; i.e., ones that are not generic but aretailor-made for the question. (A) may have seemed tempting, because it casts doubton one part of the ad, but the question is asking specifically for a choice that castsdoubt on the implication that the Zenith is of unusually impressive quality.

• Did you notice that correct answer (C) introduces a version of the “unrepresentative-sample” flaw? The group of test-drivers is not a good sample for gauging theZenith’s impressiveness, because most of them have already decided to buy aZenith. So their behavior doesn’t indicate how average, randomly-selected car-buyers would respond to the Zenith.

Page 65: 18-PrepTest 18 Explsdl.keywin.org/4/9/49d31fb5129d388a6dd62829ca482add.pdf · 2012. 4. 13. · wrong choices in a “could be false” question are statements that must be true.)

LSAT PREP _____________________________________________________________ LSAT Test XVIII Explained: Section IV

© K A P L A N 63

8. (B)

NOTE: The Law Services official answer key may read (C) for this question, but this is atypo. The correct answer is (B).

A mini-paradox of sorts—more and more people in Malsenia have become interested inclassical music and are buying classical records, but audiences at classical concerts areshrinking. The author concludes that the “new Malsenian converts to classical music” aresatisfied with recorded classical music and have no desire to hear live classicalperformances. In other words, because they don’t attend classical concerts, the authorconcludes that they don’t want to attend them. He must be assuming that they could attendthe concerts if they chose, or as (B) puts it, that they have the option of attending classicalconcerts. Otherwise, they may long to hear classical music live, but for some reason oranother (distance, time, money) are simply unable to do so.

(A) Although the author says that many buyers of classical records were drawn to thattype of music when they heard it played on television, he needn’t assume that only thosepieces actually played on television attract buyers. TV exposure to classical music helpsdevelop interest, but once people become interested in classical music, they might buy anyclassical record, TV hit or not.

(C) This choice may have been tempting, as it may seem to play into the assumption incorrect choice (B). If the number of classical concerts has indeed decreased, then wouldn’tit be logical to say that this would hinder the public’s opportunities to see classical musiclive? Possibly, but notice that the evidence specifically focuses on audience shrinkage atclassical concerts that do come off, so a denial of choice (B) hurts the argument much morethan a denial of choice (C). In other words, even if there are fewer classical concerts offered,based on the evidence presented, the conclusion that the listeners in question have nodesire to hear live performances is still valid.

(D) You may wonder: If Malsenians could hear live performances on record, wouldn’t thisdamage the argument that Malsenians have no desire to hear live performances? But wecan’t confuse the two forms of hearing live music: In this context, the author differentiatesbetween recorded classical music and live performances, so hearing a recorded liveperformance is not the same thing as attending and listening to a real live perfomance.Therefore, it wouldn’t matter if the classical records available are recordings of actualconcerts, because the evidence (that audiences at live concerts are shrinking) still supports theauthor’s conclusion in the last sentence. We deny the choice, yet the conclusion still stands,so this statement need not be assumed in the argument.

(E) It’s not necessary that the author assume that concerts include music that’s unavailableon recordings. The author’s point concerns the medium (record vs. live concert) more thanthe content (what actual material is played in each format).

• The difference between what someone does and what someone desires to do oftenplays a role on LSAT questions. The discrepancy between the two can often beresolved by the existence of an alternative explanation.

Page 66: 18-PrepTest 18 Explsdl.keywin.org/4/9/49d31fb5129d388a6dd62829ca482add.pdf · 2012. 4. 13. · wrong choices in a “could be false” question are statements that must be true.)

LSAT PREP _____________________________________________________________ LSAT Test XVIII Explained: Section IV

64 © K A P L A N

9. (C)

NOTE: The Law Services official answer key may read (B) for this question, but this is atypo. The correct answer is (C).

The author makes a familiar mistake, identified by (C). Because brain damage is found inpeople with Parkinson’s disease (i.e. there’s some “correlation” between the conditions),the author assumes that brain damage causes Parkinson’s disease. But it’s equally possiblethat Parkinson’s disease causes brain damage, or that a third agent leads to both conditions,or that the correlation is coincidental. Therefore, the author’s plan to diagnose futureParkinson’s with brain scans is plain silly.

(A) Very true—the author does fail to establish this. But then again, there’s no reason whyshe should establish this. The author claims only that brain scans can be used to diagnosethe likelihood of coming down with Parkinson’s disease, period. To make this point, shedoesn’t need to establish that brain scans are the most accurate diagnostic tool available.

(B) Treatment of Parkinson’s disease isn’t discussed at all, so she certainly hasn’toverestimated the role of brain scans in determining treatment.

(D) The author makes no assumptions about how soon people would want to know aboutthe possibility of Parkinson’s disease; she said that brain scans could be used as earlydiagnostic tools, not that people would actually use them as such.

(E) The treatment and monitoring of Parkinson’s disease are beyond the scope of theargument; the author is talking about detecting a tendency towards the disease, and needn’taddress these other subjects to prove her point.

• You should recognize a correlation when you run across one, and take someimpression of how strong it is (easier to do if you’ve read the question stem andknow you’re looking for a flaw). We’re told that this type of brain damage “is foundin people suffering from Parkinson’s disease”: it “is” found, not “is always found”or even “is usually found.” That’s not enough to build much of a conclusion on.

• When they ask for an author’s “reasoning error,” be extremely suspicious of anychoice that introduces a topic the author never discusses.

• Many wrong choices—such as (A) and (E) here—attempt to fault the author foromitting something that he or she is not logically obligated to include. When achoice begins “fails to establish that . . .” or “neglects to specify how . . .”, thequestion is not did the author in fact fail or cover or neglect these issues, butwhether or not the argument depends on the author’s doing so. Very often it doesnot.

• Beware of the Can vs. Should scope shift, or variations on it such as the one thatappears in choice (D). If an author states that something can be done, the fact thatsome think it should, or shouldn’t be done, or that some would or wouldn’t takeadvantage of it, is logically irrelevant.

Page 67: 18-PrepTest 18 Explsdl.keywin.org/4/9/49d31fb5129d388a6dd62829ca482add.pdf · 2012. 4. 13. · wrong choices in a “could be false” question are statements that must be true.)

LSAT PREP _____________________________________________________________ LSAT Test XVIII Explained: Section IV

© K A P L A N 65

10. (A)Bad news for books published in the last 150 years: they were almost all printed on acidicpaper, and acidic paper destroys itself (though the process can be slowed if the paper isstored in a cool, dry place). There’s a ray of hope for some books, in that techniques todeacidify paper are being developed, but such techniques will probably be used only on“historically significant” books. So the other books are probably out of luck, which meanswe can infer (A): If a book isn’t historically significant and was printed in the last 150 years,it will probably decay completely (since it’s almost certainly acidic and it probably won’t bedeacidified).

(B) We don’t know what proportion of books published in the last 150 years will be judged“historically significant,” so we can’t be sure that “almost all” such books will be deniedthe techniques to deacidify them.

(C) Maybe lots of books that weren’t made on acidic paper, but on some other type of self-destructive paper, are also destroying themselves. (C) entirely omits the time frame:Though “almost all” books in the last 150 years were printed on acidic paper, there maystill have been a great many books printed before that time on other paper that aregradually destroying themselves.

(D) The stimulus says that only historically significant books will be deacidified (and eventhat’s probable, not definite), which doesn’t imply that all historically significant bookswill be deacidified.

(E) If anything, the opposite of this can be inferred, since the stimulus says that being in acertain environment can slow book deterioration. It’s therefore highly unlikely that allthese books are in the same stage of deterioration.

• Qualified stimuli tend to yield qualified inferences. Notice the perfect matchbetween the “probably” in the stimulus and the “probably” in the correct choice.

11. (D)A good response to the civil libertarian’s argument should address her claim thatcensorship of offensive art is harmful to society. Instead, the censorship advocate merelysays that many people agree about what art is potentially offensive. Even if that’s true, ithas no effect on the claim that censoring such art would have negative effects on society; inother words, this rebuttal is irrelevant.

(A) The rebuttal doesn’t base a general rule on a specific case; it reaches a generalconclusion in favor of censoring potentially offensive art, based on the general claim thatmany people agree on what qualifies as such art.

(B) The rebuttal doesn’t put forth a principle of its own; if anything, it denies a principleput forth by the civil libertarian. Moreover, it’s not really basing its conclusion on acommonly held belief in the way that (B) implies; it doesn’t say “many people agree X isoffensive, therefore X is offensive.” Instead, it implies that since there’s common agreementon what is potentially offensive, censoring potentially offensive art isn’t harmful.

Page 68: 18-PrepTest 18 Explsdl.keywin.org/4/9/49d31fb5129d388a6dd62829ca482add.pdf · 2012. 4. 13. · wrong choices in a “could be false” question are statements that must be true.)

LSAT PREP _____________________________________________________________ LSAT Test XVIII Explained: Section IV

66 © K A P L A N

(C) There’s no ad hominem attack here; the rebuttal makes no reference to the libertarian’scharacter.

(E) No; actually, the rebuttal employs the same vocabulary and tone as the libertarian’soriginal statement.

• It’s not uncommon on LSAT Logical Reasoning and Reading Comprehensionquestion to get hung up trying to decide between the correct answer and onetempting wrong choice (like (B) perhaps), while the three other wrong choices areclearly no good. Cross off the far-out choices and compare the two tempting choicesclosely; evaluate each word for word if necessary. Get in the habit of recognizinghow the most tempting of wrong choices go off the rails (like (B) talking about“extracting a principle”).

• Picture these arguments being played out in real life. If you were at a cocktail party,let’s say, and happened to hear this argument and rebuttal, your most likelyresponse to the censorship advocate would be: “Huh? What does that have to dowith what she just said?” Your instincts and good common sense should tell youimmediately that the response was irrelevant to the claim being made. The onlychallenge lies in recognizing how the testmakers choose to word the correct choice.

12. (C)The only real reason the author gives for saying that Australia’s placentals are not native tothe continent is the fact that their ancestors could travel long distances across the sea; thatis, their ancestors could have made it to Australia from somewhere else. So the authorassumes that any placental’s ancestor that could have reached Australia from elsewhere, didreach Australia from elsewhere. As (C) puts it, she assumes that the only placentals thatcould possibly be natives of Australia are those whose ancestors could not have reachedthe continent from elsewhere.

(A) is irrelevant; the author is trying to establish that placentals’ ancestors came to Australiafrom elsewhere, and needn’t assume anything about where marsupials came from.

(B) The author is arguing that the placentals’ ancestors could have reached Australia undertheir own power (swimming, flying, floating); that’s why she says that they aren’t native tothe place. Therefore, she needn’t assume that humans brought them to Australia.

(D) would support an argument that marsupials are native to Australia (unless they werebrought to the continent by humans), but is irrelevant to the author’s argument thatplacentals aren’t native to Australia.

(E) is silly; the argument has no specific interest in “seals, bats, and mice” as opposed toother placentals, nor does the argument depend on whether or not Australians placentalsand marsupials frequent the same areas.

Page 69: 18-PrepTest 18 Explsdl.keywin.org/4/9/49d31fb5129d388a6dd62829ca482add.pdf · 2012. 4. 13. · wrong choices in a “could be false” question are statements that must be true.)

LSAT PREP _____________________________________________________________ LSAT Test XVIII Explained: Section IV

© K A P L A N 67

• The author never explicitly makes the point that the placentals’ ancestors could havecome to Australia “from elsewhere,” she only states that they could travel longdistances. When you read a stimulus, you have to follow the author’s train ofthought actively and ask why she introduces each point. Then you can follow themovement from evidence to conclusion, and see where they fail to connect.

• Use Keyword phrases and your understanding of common sentence structures tohelp you to locate the author’s conclusion. “It is clear, however, . . .”, which precedesthe colon, is the author’s conclusion, and what follows the colon is the evidence forit.

• Remember that an assumption is intimately related to the conclusion—it’s requiredin order for the conclusion to remain valid. So, in this case, as soon as you know thatthe conclusion centers around placentals, it should be easy to rule out the choicesthat focus solely on marsupials.

13. (A)The problem is simple: How can room air conditioners produced by Japanesemanufacturers be described as “more reliable” than those produced by United Statesmanufacturers when both types of air conditioners have the same average lifetime? (A)solves the problem by defining “reliability” as a measure of how long a product functions“without needing repair.” So it’s possible that U.S. air conditioners have an averagelifetime as long as that of Japanese air conditioners (that is, they both last about the sameamount of time before finally and completely breaking down), but U.S. air conditionersexperience minor breakdowns and have to be repaired sooner and more frequently. In thissense, the American models may very well be less reliable.

(B) is irrelevant. The term “United States manufacturers” retains the same meaning in bothstatements, so it really doesn’t matter how it’s defined. The seeming discrepancy created bythe statements holds even if U.S. manufacturers have facilities in other countries.

(C) actually advertises its own irrelevance; if the damage referred to is relatively constantfor both countries, then it can have little to do with explaining away any discrepancybetween the two countries’ products.

(D) Useless background information: We’re not interested in the history of air-conditionermanufacturing, but in how the “unreliability” of U.S. air conditioners can be reconciled withtheir healthy lifespans.

(E) gives a reason why Japanese air conditioners may be more reliable that U.S. airconditioners, but still ignores the question of why U.S. air conditioners last as long.

• When asked to reconcile two statements or resolve a paradox, you must understandexactly where the contradiction takes place. For this, paraphrasing can be veryhelpful.

Page 70: 18-PrepTest 18 Explsdl.keywin.org/4/9/49d31fb5129d388a6dd62829ca482add.pdf · 2012. 4. 13. · wrong choices in a “could be false” question are statements that must be true.)

LSAT PREP _____________________________________________________________ LSAT Test XVIII Explained: Section IV

68 © K A P L A N

• A good paradox answer choice must explain all aspects of the paradox—choices like(E), which explain only one claim without addressing the other, are common wrongchoices.

14. (D)A fairly fancy way of asking for a weakener, don’t you think?—but that’s in effect what wehave. We’re looking for a statement that suggests that the fact that the global ozone layerhas remained constant should not allay fears that polar marine life will be damaged byharmful ultraviolet radiation (thus undermining the relevance of the evidence). And weget it in (D): If atmospheric ozone has shifted from the Antarctic (one of the polar caps) toother regions, then the overall global ozone level could well remain constant while theAntarctic ozone level falls dangerously low.

(A) The fact that most animals don’t live in the Arctic or Antarctic regions (no surprisehere) doesn’t suggest that the government’s statistics about global ozone levels areirrelevant to the question of Antarctic ozone levels.

(B), (C), and (E) are all, if we may put it so, irrelevant to the question of relevance; none ofthem addresses the government officials’ statistics at all. The seasonality of the decreaseover Antarctica, what people were concerned about before 1980, and the fact that someultraviolet rays inevitably reach the earth’s surface are all things that don’t speak to theidea of constancy in atmospheric ozone. A statement can’t damage the relevance ofevidence that it ignores. (In contrast, correct choice (D) offers a scenario that explains theconstancy found in the government statistics in a way that does undermine the veryrelevance of such evidence.)

• Pay attention to geographical limits! The switch from “Antarctic” ozone in thebeginning to “global” ozone in the end is a red flag. It’s a classic scope shift, and theanswer almost has to have something to do with it.

15. (A)Here’s one you may have been able to pre-phrase: The imminent recession, despite the factthat will hurt most other businesses, is expected to help Goodbody Inc. find tenants for theParrot Quay commercial development it has just completed outside the city’s financialcenter. We’re asked to resolve the paradox of a recession helping Goodbody’s search fortenants. (A) explains that the financial crunch caused by the recession will make businesseswant to take advantage of lower rents available in areas outside the financial center, whichplays into Goodbody’s hands by making its new office space especially desirable.

(B) This link to the financial center is a nice amenity, but is certainly no reason why arecession would actually work in Goodbody’s favor.

(C) offers little more than irrelevant background information; it also has little to do withthe question of why a recession will help the Parrot Quay development. In fact, if during aprevious recession businesses in the area went belly up, this certainly doesn’t bode verywell for the development in the new recession.

Page 71: 18-PrepTest 18 Explsdl.keywin.org/4/9/49d31fb5129d388a6dd62829ca482add.pdf · 2012. 4. 13. · wrong choices in a “could be false” question are statements that must be true.)

LSAT PREP _____________________________________________________________ LSAT Test XVIII Explained: Section IV

© K A P L A N 69

(D) is nothing but bad news. It tells us how much Goodbody will suffer in its financialcenter holdings if the recession gets bad enough. That certainly doesn’t explain how thecoming recession will help Goodbody in its quest for tenants. (It’s way too much of a leap tosay that these tenants will flock to populate the new development; we simply can’t inferthis.)

(E) may be able to help explain why the recession might not hurt real estate developers asmuch as we’d think: It will have its worst effect on businesses that aren’t the biggest realestate customers. But this still doesn’t explain why the recession will actually helpGoodbody find tenants.

• With practice you can learn to recognize important details as they’re presented in thestimulus. For instance, an experienced test-taker reading the description of the newdevelopment, will key on the idea that it’s “outside the financial center”; the locationof a real-estate development is likely to be a key to its success.

• Stay away from “neutral” choices in paradox questions—a choice that’s notinconsistent with the stimulus, but simply doesn’t actually resolve the discrepancy.In this case, a neutral choice may hint at a reason why the recession might not hurtGoodbody, but the right choice must go much further—it has to explain how therecession will actuallyhelp Goodbody.

16. (C)The key here is to recognize that Kim’s reason for saying the use of electronic fetal monitorsshould be discontinued is that they are more intrusive than ordinary stethescopes withoutimproving the chances of a healthy baby being born. So the principle that backs this upwill have to hit on this “intrusion” issue—it forms the crux of Kim’s argument. Kim’scontention would be best supported by (C); there are two methods of monitoring fetalheartbeat available to hospitals, both provide the same kind of information (as evidencedboth by Anders’ comment and by Kim’s comment that neither method is better at servingbabies), so the more intrusive method (fetal monitors) should be discontinued.

(A) Neither Kim nor Anders says that ordinary stethescopes provide more informationthan fetal monitors, nor do we get the feeling that the stethescope came along after theelectronic monitoring, so (C) wouldn’t support Kim’s recommendation to abandon themonitors.

(B) misses the fact that Kim is making a choice between two procedures, and we need aprinciple that supports his choice; since electronic fetal monitors presumably do provideuseful information (as useful as the information provided by stethescopes), (B) gives noreason to discontinue their use.

(D) would support Anders’ argument rather than Kim’s.

(E) refers to considerations that have no place in Kim’s argument, so it doesn’t support thatargument—Kim never advocates “periodically re-evaluating” procedures and neverquestions the reliability of the information provided by fetal monitors.

Page 72: 18-PrepTest 18 Explsdl.keywin.org/4/9/49d31fb5129d388a6dd62829ca482add.pdf · 2012. 4. 13. · wrong choices in a “could be false” question are statements that must be true.)

LSAT PREP _____________________________________________________________ LSAT Test XVIII Explained: Section IV

70 © K A P L A N

• You may have tried reading only Kim’s argument to pick up this point, which isn’t abad idea considering the stem tells us that’s who we’re interested in. The problem inthis case is that Kim never mentions the notion of equivalent information; the closesthe comes is to say both methods provide the same chances of delivering a healthybaby (which, since we’re comparing monitoring procedures, means virtually thesame thing). If you only read Kim’s argument and found yourself uncertain, that wasthe time to glance at Anders’ argument.

• Despite the possible snag mentioned above, the fact that (C) is the only choice thatexplicitly deals with a choice between two methods—the cornerstone of Kim’sargument—should have made it the clear front-runner.

17. (A)Notice that Kim isn’t arguing that developing and using the monitor was a waste of timeand money; he’s arguing that there’s no longer any reason to continue using the monitor inview of the fact that there’s a less intrusive alternative. Anders’ reply that the expenditurefor electronic monitoring in the past has been worth it, because the monitor taught doctorswhat to listen for with the stethescope, is all well and good, but it doesn’t address the issueof continuing to use the electroninc montiors. The benefit Anders speaks of has alreadybeen gained, and cannot be used to argue for the continuation of this method. (A) says justthat, in more general terms.

(B) The problem isn’t that Anders assumes what he sets out to prove (circular reasoning),but that he sets out to prove the wrong thing (that the monitor was well worth the moneyspent on it in the past).

(C) Anders never confuses high quality with high cost; he never even discusses the“quality” of the monitor. All he says that it aided doctors by telling them what to listen forwith the stethescope.

(D) Anders doesn’t defend the monitor because it’s technologically advanced, but becausein the past it helped doctors learn how best to use the stethescope.

(E) Anders doesn’t argue in favor of the monitor procedure because it is widely used, nordoes he claim that it’s the best procedure available; the whole problem about Anders’argument is that it ignores the question of whether or not the procedure should continue tobe used.

• When you’re asked to critique a reply to an initial argument, it can help toparaphrase the first argument’s conclusion (or simply to get a clear idea of theconclusion in your head), and then see if the reply fails in some obvious way toaddress that conclusion.

• Sometimes scope shifts involve shifts in the time frame. Here, Anders argues for thecontinued future use of a technology based on a past benefit of the technology that’sno longer relevant.

Page 73: 18-PrepTest 18 Explsdl.keywin.org/4/9/49d31fb5129d388a6dd62829ca482add.pdf · 2012. 4. 13. · wrong choices in a “could be false” question are statements that must be true.)

LSAT PREP _____________________________________________________________ LSAT Test XVIII Explained: Section IV

© K A P L A N 71

18. (D)The author criticizes Professor Hartley as intellectually dishonest because her bookcontains unattributed passages that can be found in an earlier book by Professor Lawrence.This may sound like a just criticism until you realize that the author assumes that eitherLawrence’s book, or some other work used by Lawrence’s book, is the source of the ideasand passages in question, and that Hartley borrowed (or stole) the ideas and passages fromthat source. Specifically, as (D) puts it, the author assumes that Hartley herself wasn’tLawrence’s source for the ideas or their wording; after all, it’s possible that Lawrenceborrowed the passages from earlier works of Hartley, or conversations with her, or a coursetaught by her. If (D) is false, and Lawrence did borrow the passages from Hartley, thenobviously there’s no need for Hartley to attribute the passages to anyone else.

(A) The author never delves into the importance of the passages to Hartley’s book; hefocuses only on the issue of Hartley not attributing part of the work to someone else. Wehave no idea how important the author feels these passages are to the overall work, so wecan’t say that the author assumes that the book wasn’t doable without them.

(B) It’s not necessary that Hartley actually had access to Lawrence’s book as she wrote; shemight have been writing from notes. More importantly (and much more subtly, as well),the author never says that Hartley got the passages directly from Lawrence and shouldhave attributed them to Lawrence; she may have gotten them from an earlier source alsoused by Lawrence, or from an interim source that relied upon Lawrence’s book.

(C) The author never claimed that the passages in question didn’t represent Hartley’sconvictions, so (C) isn’t relevant to his argument and needn’t be assumed.

(E) The author isn’t interested in Hartley’s feelings towards the passage, and needn’tassume anything about it; his only point is that Hartley did use the passages, they weresomeone else’s work (that’s the assumption in (D)), and she should have attributed thepassages to their ultimate source.

• Once again, the answer to an LSAT question hinges on whether or not the test-takerunderstands that the chain of causation could go both ways (Lawrence to Hartley orHartley to Lawrence). That point is camouflaged well in this rather clever question.Get into the habit of thinking flexibly about the chain of causation withinarguments, and questions like this will be a breeze.

19. (B)Sounds like a pretty sound principle: If someone gives you a piece of unsolicited(unasked-for) advice, and that advice just happens to be something that would help theperson giving it, don’t be too ready to follow the advice unless you know it would helpyou too. Which choice properly applies the principle? In (B), the salesperson comes up toRamon and volunteers her opinion—that’s the unsolicited advice. She warns against theleast expensive model; this advice would benefit her, since her commission will be bigger ifRamon rejects the least expensive model and goes for something more expensive. (B)judges that on the advice of the salesperson alone (i.e. without finding out whether adifferent model would prove better for him), Ramon shouldn’t automatically reject theleast expensive model—a perfect application of our principle.

Page 74: 18-PrepTest 18 Explsdl.keywin.org/4/9/49d31fb5129d388a6dd62829ca482add.pdf · 2012. 4. 13. · wrong choices in a “could be false” question are statements that must be true.)

LSAT PREP _____________________________________________________________ LSAT Test XVIII Explained: Section IV

72 © K A P L A N

(A) goes too far and says that Harriet should wholly reject Floyd’s advice; the stimulusonly said one should regard such proferred advice “with skepticism” in the absence offurther information.

(C) The advice here isn’t unsolicited—Mario asks for Yvette’s opinion, and that’s enoughto kill it right there.

(D) For one thing, this advice benefits the editor, and there’s no third party as part of theprinciple. The only way Sara’s advice can be said to benefit her is that it will help sales of atextbook that she helped write. But Ron helped write it too, so the advice will be as helpfulto him as it is to her. Too many problems, so the principle doesn’t apply.

(E) gives us a situation where the person giving advice is actually hurting his own interestsin order to advance the interests of the person he’s advising. Noble behavior, but notcovered by the principle.

• In order for the judgment recommended in a choice to be justified by the stimulusprinciple, the situation described in that choice must correspond exactly to thesituation covered by the principle. If there are three parts to the principle, all threeparts must be covered in a specific case in order for it to conform. If even one part ismissing, the game is up. Therefore, . . .

• The best way to check the choices is to see if the important features of the stimulussituation are represented (unsolicited advice, adviser’s interest vs. advisee’s interest,regard advice with skepticism). This is almost like the method for Logic Gamesacceptability questions—check the choices against the “rules.” No “unsolicitedadvice?” Then cross that choice off right there.

• These questions are also rather like Parallel Reasoning questions. If you workcleverly to take the choices apart, matching them up with the elements of thestimulus like we teach in Parallel Reasoning, you’ll find that you need not read everychoice meticulously in order to cross them off.

20. (A)In the abstract, the argument goes like so: A statement of fact speaks to an entity’s weakness.A general principle is stated, suggesting that a business that has failed should beterminated. An exception is noted: This particular entity doesn’t fall under the generalprinciple. A conclusion is reached: The entity should not be terminated, at least on thebasis of this one weakness. Even though (A) presents the pieces in a different order—principle, statement of fact, conclusion, exception—all of those pieces do match up.(Actually, the moment you saw that only (A)’s conclusion matches up to that of thestimulus—”X is an insufficient reason to shut down Y”—you could’ve stopped rightthere!)

(B) is just a justification for not holding the OKESA company to the same tough standardsas other, similar businesses; it’s not attempting to deter punishment for some sincommitted by OKESA.

Page 75: 18-PrepTest 18 Explsdl.keywin.org/4/9/49d31fb5129d388a6dd62829ca482add.pdf · 2012. 4. 13. · wrong choices in a “could be false” question are statements that must be true.)

LSAT PREP _____________________________________________________________ LSAT Test XVIII Explained: Section IV

© K A P L A N 73

(C) argues that an all-encompassing anti-gambling law should be thought of as covering atype of betting not mentioned in that law. Very few parallels to the stimulus whatsoeverhere.

(D)’s argument that an exceptional entity (cockatiels) should be treated like everyone elsewould be a better answer if the stimulus were arguing for the county parks to be closed. Butwe need a choice that argues against using a general principle to punish an entity.

(E) sets up a tidy parallel (since youths aren’t subject to the same laws, they shouldn’t getthe same punishments) that might look superficially similar to the stimulus, but it lacks asingle concrete case parallel to the county park (or the prime-time TV series) and missesthe central theme of “inadequate justification” in its conclusion.

• Be acutely aware of the testmakers’ tendency to mix up the order of evidence andconclusion in a Parallel Reasoning correct answer. Deciding which piece of theargument to mention first is a stylistic decision, not a logical one. The LSAT writerswant to see whether you recognize that fact, so they often switch terms around in theright answer as they do here. Watch for it!

21. (E)This is an odd exchange: Mark doesn’t seem to be listening to Jane very carefully. Jane saysthat Professor Harper’s ideas about modifying guitars are useless, because there’s nogeneral agreement about what guitars are supposed to sound like or how their soundshould be evaluated. Mark agrees in rejecting Harper’s ideas, but his reason for doing so isthat if Harper’s ideas really resulted in superior sound, they would have been adoptedalready, since it only took a few years for Torres guitars to be adopted due to theirsuperior tonal quality. In other words, Jane’s argument is based on the idea that there’s noagreement on what superior guitar sound is; Mark’s argument is based on the idea thatsuperior sound is quickly and generally recognized. As (E) says, the arguments are basedon conflicting suppositions.

(A) The last thing Mark is doing is trying to correct Jane’s reasoning—he’s actuallyattempting (albeit feebly) to agree with her assertion.

(B) As we’ve seen, the two arguments have contradictory premises; what they share is theconclusion that Harper’s ideas aren’t useful.

(C) is the opposite of the case; Mark and Jane reach the same conclusion (that Harper’sideas are of no value), but use two very different routes to get there.

(D) Far from restating Jane’s argument, Mark’s argument relies on premises that directlycontradict Jane’s reasoning. If Mark’s argument is accurate, Jane’s premises must be false.

• The habit of reading the question stem first can save valuable seconds on anunusual question like this, by telling you what to look for when you read the twoarguments. That edge is especially welcome at the very end of the test when you needto economize on time and energy and can’t afford any wasted effort.

Page 76: 18-PrepTest 18 Explsdl.keywin.org/4/9/49d31fb5129d388a6dd62829ca482add.pdf · 2012. 4. 13. · wrong choices in a “could be false” question are statements that must be true.)

LSAT PREP _____________________________________________________________ LSAT Test XVIII Explained: Section IV

74 © K A P L A N

22. (E)Once again, causation rears its ugly head. The author cites a correlation discoveredbetween tinted eyeglasses and conditions like depression and concludes that conditionslike depression lead people to put on tinted eyeglasses. Clearly he’s assuming that thecausation doesn’t run the other way—as (E) points out, the author must assume thatwearing tinted glasses doesn’t lead the wearer to conditions like depression.

(A) and (C) The author concluded that depression was one of the causes of wearing tintedglasses, but he didn’t give any opinion about the causes of depression, so he needn’tassume anything about those causes.

(B) It’s entirely possible that depressed and hypochondriacal people might want todistance themselves from other people and use glasses as a means of doing so; in fact, theauthor suggests something of the sort. Negate (B), and you don’t hurt the argument at all,which means that (B) is not assumed.

(D) The author offers his own tentative explanation for why psychologically troubledpeople might wear tinted glasses, but it isn’t central to his argument. And this purpose forthe glasses has nothing to do with signaling others, so this need not be assumed for theauthor’s argument to stand.

• A couple of Keywords tell you that the author’s theory about why “people whoserelationship to the world is psychologically painful” wear tinted glasses is only aspeculation, and not central to the argument: the “perhaps” before he states thetheory, and the “at any rate” after it. “At any rate” means that, true or false, it won’taffect the conclusion.

• Whenever one thing is said to cause another, ask yourself if it’s possible to reversethe chain of causation. This is especially useful to look for in Assumption, FaultyLogic, and Strengthen/Weaken questions.

23. (B)The author’s argument, in its essentials, runs like this: Tests run on British hospital patientsshow a correlation between wearing tinted glasses and having complaints like heart painand digestive distress, therefore people wear tinted glasses because they have a tendencytowards depression and hypochondria. The only choice that wouldn’t weaken thisargument is (B), which doesn’t even mention the question of why people wear tintedglasses, which was the whole focus of the argument. The fact that depressed andhypochondriacal people may have “valid medical complaints” doesn’t affect the author’sconclusion that people wear tinted glasses because of a tendency toward these conditions.

(A) gives an alternate reason for people to wear tinted glasses, which weakens the claimthat such glasses are worn because of the wearer’s tendency to depression andhypochondria.

(C) points out a flaw you mightn’t have noticed; all the evidence comes from Britain,whereas the conclusion is about people in general. If, as (C) says, the quality of light isdifferent in other places, there is reason to doubt the validity of the general conclusion.

Page 77: 18-PrepTest 18 Explsdl.keywin.org/4/9/49d31fb5129d388a6dd62829ca482add.pdf · 2012. 4. 13. · wrong choices in a “could be false” question are statements that must be true.)

LSAT PREP _____________________________________________________________ LSAT Test XVIII Explained: Section IV

© K A P L A N 75

(D) also strongly implies that the situation in Britain might not apply in the rest of theworld: Around the world, the dictates of fashion play varying roles in determining whowears tinted glasses. Maybe in some places all well-dressed people wear tinted glasses.

(E) undermines the validity of the conclusion by attacking the completeness of theevidence offered. The author only reported on tests given to people with “ambiguousailments.” When the sample is broadened to people with other ailments, the relationshipdescribed by the author disappears.

• This question serves as a mini-textbook of the classic ways an argument can beweakened: alternate explanation (A), geographical scope shift (C), alternateexplanation combined with a geographical scope shift (D), and unrepresentativesample (E). Remember, the only reason you go over these actual tests at all is to learneverything you can about how the testmakers think, the repetitive patterns of rightand wrong answers, and what you can expect on test day.

24. (D)The evidence suggests that nonhuman predators got the first crack at the large animals, andhumans only got to the carcasses after the other predators were finished. The statementmost supported by this evidence is (D): Humans didn’t hunt large animals themselves, butscavenged on what was left over by other predators.

(A) Even though the evidence suggests that early humans didn’t hunt large prey, we haveno reason to think that they did hunt small prey—maybe they didn’t hunt at all, but wereexclusively scavengers and plant-gatherers.

(B) We have no evidence that humans ate fruits and roots as well as the meat they nibbledfrom large carcasses; chances are they ate some kind of vegetables (grasses, leafy vegetables,whatever) but we can’t even be sure of that.

(C) is way off. There’s no basis for concluding what the most efficient method of huntingwould have been; we don’t even know how humans did hunt (or if they did).

(E) Though we know that early humans occasionally got to gnaw the less meaty parts oflarge animals, we have no reason to conclude that they spent their lives following suchprey around.

• Absolute statements are always especially important when you’re trying to draw aconclusion. “Predators that hunted large prey invariably ate the meatiest part of thecarcasses”; that means “if humans had hunted large prey, they would have eaten themeatiest parts of the carcasses.” By the contrapositive, since they didn’t eat themeatiest parts, they didn’t hunt large prey.

• Even if you’re unsure of an answer, you can often get it by throwing out the wrongchoices. Work on developing the skill of recognizing what you don’t know—this willhelp you quickly eliminate choices like the four wrong choices here which aresimply unknowable based on the information given.

Page 78: 18-PrepTest 18 Explsdl.keywin.org/4/9/49d31fb5129d388a6dd62829ca482add.pdf · 2012. 4. 13. · wrong choices in a “could be false” question are statements that must be true.)

LSAT PREP _____________________________________________________________ LSAT Test XVIII Explained: Section IV

76 © K A P L A N

25. (E)Ursula does most of the work for us. George rejects the educator’s claim that when peopleare read to as children, they’re more likely to enjoy reading as adults, based solely on thecounterexamples of himself and his cousin Emory. Ursula objects that this counterexampleproves nothing; it would be effective if the educator had claimed that all adults who enjoyreading had been read to as kids, but the educator didn’t make “that sort of claim.” This is(E)’s point: the educator’s claim is consistent with the existence of “nonconforming cases”or counterexamples. The educator said that children who are read to are “more likely” toenjoy reading when they grow up; this leaves open the possibility that some people (likeEmory) are read to and don’t enjoy reading as adults, and others (like George) aren’t readto, but do enjoy reading as adults.

(A) George doesn’t compare his family’s experience to other people’s experience; he useshis family’s experience to dispute the educator’s theory.

(B) There’s no need for George to go into these details, since the educator’s theorypurported to hold for books in general, and didn’t touch on these details either.

(C) The educator’s theory doesn’t differentiate between different types of reading on thebasis of how “relaxing” they are, and there’s no reason George should either. Theeducator’s theory concerns the general effect of being read to as a child.

(D) There’s no need for George to show that the educator’s position is held by the majorityof educators; he’s interested in refuting the claim, and for that purpose it doesn’t matter ifthe claim is made by one educator, most educators, or all educators.

• Both (B) and (C) attack George for not making what is an unnecessary and irrelevantdistinction; irrelevant distinctions also show up frequently as wrong choices in“Weaken the Argument” questions.

• You may have spotted the flaw wihout even reading Ursula’s rebuttal. The questionstem tells you to focus on George’s reasoning, so you may have been able to find theanswer from his argument alone. If not, reading Ursula’s argument proves quitehelpful in that it pinpoints the flaw for us.

Page 79: 18-PrepTest 18 Explsdl.keywin.org/4/9/49d31fb5129d388a6dd62829ca482add.pdf · 2012. 4. 13. · wrong choices in a “could be false” question are statements that must be true.)

I.N. LL3110 Rev.A Printed in the USA